You are on page 1of 61

Romanian Mathematical

Competitions

RMC 2004

EDITORS
Mircea Becheanu
Radu Gologan

Romanian Mathematical Society & Theta Foundation


Romanian Mathematical
Competitions

RMC 2004

EDITORS
Mircea Becheanu
Radu Gologan

Romanian Mathematical Society & Theta Foundation


Romanian Mathematical
Competitions

RMC 2004

EDITORS
Mircea Becheanu
Radu Gologan

CONTRIBUTORS
Marian Andronache
Bogdan Enescu
Mihai Baluna
Cdlin Popescu
Dinu Serbanescu
Dan Schwarz
ISBN 973-0-03572-5

ROMANIAN MATHEMATICAL COMPETITIONS

CONTRIBUTORS:
MARIAN ANDRONACHE
National College “SE. Sava” Bucharest,
BoGDAN ENEscU
National College “B.P. Hasden Buziu It became a tradition to publish the most interesting problems given at the
Mumal BXLUNK Romanian Mathematical Olympiad and some other competitions for High School
National College “Mihai Viteazul” Bucharest Students. consists of two
MircEa BECHEANU - Editor The present booklet is the eleventh in the series. Thethebook District and Final
parts. First part presents all the proposed problems for
Department of Mathematics, University of Bucharest. Younds of the National Mathematical Olympiad together with the problems given
RADU GoLOGAN - Editor for the qualification tests of the Romanian Teams participating in the Inerna-
Institute of Mathematics and tional Mathematical Olympiad and the Junior and Senior Balkan Competitions ,
University “Politehnica” Bucharest regional Romanian Con-
respectively, We also gathered some problems from otherdiscussed
tests and made a selection from the Shortlist that we during different
CRLIN PopEscy rounds of the Olympiad.
Institute of Mathematics Most of the problems are new or, to our knowledge, do ot have equivalent
DNy $ERBANESCU statements in the mathematical olympiad literature. We have to thank to the large
National College “Sf. Sava” Bucharest. number of teachers in mathematics, mathematicians and students who contributed
during the year with more than a thousand of problems such that our selection
DAN SCHWARZ Drocess was not easy. Some of the problems came from different other sources:
Board for the National Mathematical Olympiad Shortlisted problems from the IMO and BMO, various mathematical journals, or
oven from some of the large variety of Internet sources one hes access now-days
The text is a joint work of the editors and the contributors. main sponsors of
We would like to thank SOFTWIN and Medicoverthe involvement in the
Copyright © 2004 Societatea de Stiinte Matematice din Roménia the Romanian IMO Team who, beside some otherbook. important
Olympiads, supported us in part to publish theThanks areThanks are due to "Gill
Al rights reserved. No part of this publication may be reproduced or distributed in any Publisher” and to the "Sigma Foundation®. also duc to Luminita
form or by any means, or stored in data base or a retrieval system, without the prior Staf and to Barbara Ioneseu from "Theta for helping the editors in the process
written permission of the publisher. of producing this bookle.
The Jast but not the least, wo are grateful to the Board oftechnical the Institute of
constant
Mathematics “Simion Stoilow”, in Bucharest, for theseminars support
Sociotatea de tiinte Matematice din Romania in the Mathematical Olympiads and in the training for students.
Str. Academici 14, 70103 Bucuresti, Roménia
E-mail: 0ffice0rns . unibuc.ro
Home page: hitp:/ /www ssmro.com
tel/fax: +40(21)3124072
‘Typed and technically edited by Theta Bucharest, Mircea Becheanu
hutps/ fwwsr theta.ro June 22, 2004 Radu Gologan
Printed at NECOZ PREST
Str. Oracolului 8, Bucuresti, tel. -+40(21)6106328
CONTENTS

FIRST PART Proposed Problems


L1 District Round
1.2, Final Round ... .
13 Selection Tests for the d MO R
1.4. Selection Tests for the Junior Balkan Mathematical Olympiad .
L5. Shortlisted Problems for the National Mathematical Olympiad .
1.6. Regional Mathematical Competitions — Selected Problems ...

SECOND PART Proposed Problems with Solutions


2.1. District Round ... .
2.2, Final Round ... JRT PP
2.3. Selection Tests for the BMO and IMO . . 6
2.4. Selection Tests for the Junior Balkan Mathematical Olympiad ... 82
2.5. Regional Mathematical Competitions ~ Selected Problems ........ 92
PART ONE

PROPOSED PROBLEMS
1.1. THE NATIONAL MATHEMATICAL OLYMPIAD
District round — March 16 2004

7" GRADE
ProBLEM 1. Find the mumber of positive 6 digt integers, such that the sum,
of their digits is 9, and four of its digits are 1,0,0,4
Lucian Dragomir
PROBLEM 2. Let D be a point on the side BC of a given triangle ABC. The
bisector lines of the angles £ ADB and ZADC intersect AB and AC at M and N,
respectively, and the bisector lines of the angles ZABD and £ACD intersect DM
and DN at K and L, respectively.
Prove that AM = AN if and ouly if MN and KL are parallcl.
Bogdan Enescu

PROBLEM 3. One considers the set A = {n eN1<y/1+vi< 2}


2) Find the set A.
b) Find the set of numbers 1 € A such that
-i <1t

PROBLEM 4. In the triangle ABC we have AB = AC and the points M, P


on AB such that AM = BP. Let D be the midpoint of BC, and let R on CM
and Q on BC, be such that A, R, Q are collinear and the line AQ is perpendicular
on CM.
Prove that:
a) ZAQC = LPQB;
b) £DRQ = 45°.
Manuela Prajea
4 PROPOSED PROSLEMS Districr Rounn 5
8" GRADE ProBLEM 2. Find the cartesian coordinates of the vertices 4, B,C od a
triangle ABC whose the orthocenter is H(~3,10), the circumcenter is O(~2, ~3),
PROBLEM 1. We say that the real numbersa si b have property P if: a?+b € and the midpoint of BC is D(1,3)
Qand P +acQ. Gabriel Popa, Mihai Balani
Prove that:
) The numbers a = # s i b,Tfi are irrational and have PROBLEM 3. a) Prove that there are infinitely many rational positive num-
bers z such that:
property P; {z%} + {2} = 0,99.
b) Ifa,bhave property P and a+b € Q\{L}, then a s b are rational numbers; b) Prove that there are no rational numbers z > 0 such that:
©) Ifa,b have property P and 3 € @ then @ 5i b are rational numbers.
)+ fa) =1
PROBLEM 2. The real numbers a,b,¢,d satisfy @ > b > ¢ > d and Bogdan Enescu
a+btctd=2004 §i a® - b?+ - d® = 2004, PROBLEM 4. A rectangle 2 x 4 s divided in 8 squares of side 1. Call M the
set of the 15 vertices thus obtained.
Answer, with proof, to the following questions: Find the points A € M satisfying: the set M\ {A} can be arnanged in pairs
a) What is the smallest possible value of a?
(A1, B1), (A2, Ba), .., (A7, By) such that
b) What is the number of possible values of a?
Mircea Fianu
KB+ AB+-+
A7Br = T.
PROBLEM 3. It s said that a set of three different numbers is an arithmetical Mihai Baluni
set if one of the three numbers is the average of the other two. Consider the set
An={1,2,...,n}, where n is & positive infeger, n > 3.
a) How many arithmetical sets are in A7
b) Find the smallest n, such that, the number of arithmetical sets in A, is 10" GRADE
greater than 2004
Lucian Dragomir PROBLEM 1. Given a positive integer n, n > 3, find the number of arith-
metical sets of 3 elements, contained in the st {1,2,...,n}.
ProBLEM 4. In a given trapezoid ABCD, let AB||CD, (B = 90° and
AB = 2DC. Consider points N and P on the same part of the planc (ABC)
such that PA and ND are perpendicular on the trapezoid’s plane, and ND ProBLEM 2. Find integers n, n > 3, having the property: there are distinct
aP=2 (a>0). If M is the midpoint of BC and the triangle MNP is equilateral, integers a1,az, .., an, such that:
find: ail-az!
) The cosine of the angle between planes (MNP) and (ABC);
b) The distance from D to the plane (MNP).
Giannina Busuioc, Niculai Solomon Bogdan Eneseu
PROBLEM 3. In the thetrahedron ABCD consider the midpoints M,N,
P,Q of sides AB,CD, AC, and BD, respectively. Prove that MN is perpen-
o' GRADE dicular to both AB and CD, and PQ is perpendicular to both AC si BD, if and
only if AB=CD, BC = DA si AC = BD,
ProBLeM 1. Real numbers a,b,c satisfy a? + 2 +¢* = 3. Prove the inequa- Radu Gologan
lity: PROBLEM 4. Let 2,y € (0,7/2). Prove that if the equality
Jal +[6] + |e] - abe < 4
Virgil Nicula (cosz +isiny)"
= cosnz +isinny
6 Proposen PronLaxs pre—— 7
is true for two consecutive integers, than it is true for al integers n. PROBLEM 2. Prove that the only continuous functions f : [0,1] — R, such
Dinu Serbiinescu that . A
/o s@e = [ gz
for any non-derivable continuous function - [0,1) — R, are the constant functions.
11" GRADE Mihai Piticari
PROBLEM 1. Let o > 0 and, for any positive integer n, consider Zns1 = Probueu 3. The ring A satisfies the following propertics:
i) s unit 14 has order p, a prime number;
=1 i) There is a set B C A containing p elements, such that: for any z,y € A,
there is b € B satisfying 7y = bya.
Pnd )l i ) lim 73 Bogdan Eneseu
Prove that A is commutative.
Ton Savu
PROBLEM 4. Let a,b € (0,1) and let £ : 0,1] — R be a continuous function,
PROBLEM 2. Consider complex non-zero numbers z1,2,..., 20, 1 > 3, such that
such that [z1] = |z2] =+ = |znss] and argz; > - > arg(znea). Define for
i,j € {L2,...in}: by = |2 = zjsnl, and let B = (b;) € M. Prove that . . -
detB =0, / f(z)dz:/ I(l)dH»/ F(Oet, for anyz < [0,1]
Clin Popescu
o o
a) Prove that a + b < 1 implies f = 0.
PROBLEM 3. A function f : R — R is said to have property P if: for every b) Prove that a + b= 1 implies f is constaut,
abe R e have f(237) € (@), 7O} Dan Marinescu
4) Give an example of a non-constant function possessing .
b) It / has P and is continuous, prove that it is constant.
Dan Marinescu
PROBLEM 4. The matrix A = (ai;) € M,(C) is defined by a3y = azg =
= ap-1,, =1 and aij =0, for the remaining set of indices (i, ).
Prove that there are no non-zero matrices B,C € Mn(C) such that
(I + A)" = B" +C", for all non-negative integer n.
Ton Savu

12" GRADE

PROBLEM 1. Let n > 2 be an integer and r € {1,2,...,n}. Consider the set


So= (A& M(Z) [rank A — r}.
a) Prove that for any A € S, and B€ S,, ABisin S;
b) Caleulate Y X,
P
Mihai Fulger, Valentin Vornicu
it Rovnp 9
8" GRADE
PrOBLEM 1. Find all non-negative integers n such that there are integers a
and b with the property:
1.2. THE NATIONAL MATHEMATICAL OLYMPIAD a+b and n®=a?+p?
Final Round — Deva, April 5, 2004
Lucian Dragomir
PROBLEM 2. Prove that the equation
7" GRADE 2yl 42t
PROBLEM 1. On the sides AB and AD of the rhombus ABCD consider the where 0 <z <y < 2 < ¢, has exactly two solutions in the set of integers.
points & and F respectively, such that AE = DF. Lines BC and DE intersect at Mihai Balun
P, and lines CD and BF intersect at Q. Prove that:
PE_QF PROBLEM 3. Consider a frustum of a regular quadrilateral pyramid
Y pptos =l ABCDA'B'C'D' in which the lines BC' are DA’ orthogonal.
b) Points P, A, Q are collinear.
2) Prove that the angle between the lines AB' and DA’ equals 60°.
Virginia Tic# and Vasile Tici b) Assume that the projection of B' onto the plane (ABC) is the incenter
PROBLEM 2. The side-lengths of a triangle are a,b,c. {he trangle ABC. Provethat the distance between the lincs G5 and AD equalsof
a) Prove that there is a triangle with sides v/a, v5 si /c. iBC".
b) Prove that ci vab+ vhbe 1 yac < a + b+ ¢ < 2v/ab + 2v/be + 2,/az. Mircea Fianu
PROBLEM 4. A cube of side 6 contains 1001 unit cubes with sides parallel
ProBLEM 3. The diagonals of the trapezoid ABCD are perpendicular and to those of the given one. Prove that one can find two unit cubes, such that the
intersect in O. Angle A equals %0°, and AB||CD, AB > CD. The diagonals center of one of them is inside or on the faces of the other.
intersect at O. OF is the bisector line of the angle AOD, E is on the segment AD, Dinu Serbiinescu
and F on BC, such that EF | AB. Denote by P respectively Q, the intersection
points of the segment EF with the diagonals AC and BD, respectively. Prove
that:
) EP = QF; 9" GRADE
b) EF = AD.
Claudiu-Stefan Popa
PROBLEM 1. Find all increasingfunctions / : {1,2,....,10} - {1,2,..., 100},
PROBLEM 4. Sixteen points are placed in the centers of a 4 x 4 chess table having the property: a-+y is a divisorof o (z)+y/(y), for any z,y € {L,2,...,10}.
in the following way: Cristinel Mortici
PROBLEM 2. For any positive integer n denote by P(n) the number of
quadratic functions f : R — B, f(z) = az® + bz + ¢, having the properties:
a)a,bce (1,20}
) Prove that may choose 6 points such that no isosceles triangle can be b) The equation /(z) = 0 has integer roots.
drawn with vertices at these points Prove that n < P(n) < 72, for all n > 4.
b) Prove that one cannot, choose 7 points with the above property. Laurentiu Panaitopol
Radu Gologan, Dinu Serbinescu
10 Proposep ProaLens FivaL Rovso 1
PROBLEM 3. Let H be the orthocenter of the acute triangle ABC, and let 11 GRADE
B',C" be the projection of the points B and C onto AC and AB, respectively. A
variable line d, passing through H, intersects the segments BC’ and CB’ in M PrOBLEM 1, Consider an integer n > 3 and the parabola of equation y* =
and N, respectively. The perpendiculars in M and N on d, intersect BB' and 2pz, with focus F. A regular n-gone AyAz -+ Ay has center at F and no one of
©C’, in P and Q, respectively. its vertices lies on the z axis. The rays FAy, FAa, .., FA, cut the parabola at
d the locus of the midpoint of the segment PQ. points Bi, By,..., Bn.
Gheorghe Szdlosy Prove that By + FBy + -+ FBy > np.
Calin Popescu
PROBLEM 4. Let p,q be positive integers, p > 2, g > 2. A set X has
property (S) if, by definition, for any p subsets B C X, i = 1,2,....p, not ProBLEM 2. Consider an integer 1, n.3 2.
necessarily different, any of them having g elements, there is a set Y C X having a) Prove that there are matrices A, B € My(C) such that
P eloments, such that the intersection of ¥ with each By, i = 1,2,...,p, has at
most one element. Prove that: rang (AB) - rang (BA) %] .
&) Any set X with pg — g clements does not satisfy property (S);
b) Any set X with pg — g+ 1 has property (S). b) Prove that for any X,Y € My(€) we have
Dan Schwarz
rang (XY) - rang (YX) < [g]

10 GRADE Lon Sava


PROBLEM 1. Let f : R — R be a function, such that |(z) ~ f(y)] < |z~ yl, ProbLem 3. Let £ : (a,5) — B be a function with the property that for any
for any real z,y. Prove that if the sequence z, (z), f(f(a)),.. is in arithmetical 2 € (a,b), there Is a non-trivial interval [az, b}, @ < 6z < @ < by < b, such that f
set, for any real 7, then there is a real a such that f(z) = + a, for any real s constanton [az, bs).
a) Prove that the image of f is a set that is finite or countable.
number z. b) Find all continuous functions that satisfy the given property.
Mihai Piticari
PROBLEM 2. Prove that a thetracdron in which pairs of opposite sides are ProsLEM 4. a) Construct a fanction f : B — Ry with the following prop-
equal and make equal angles, is regular. exty, called P:
Mircea Becheanu, Bogdan Enescu “Any z € Q is a local strictly minimum point for f*.
b) Construct f : @ — R, with the property that any point is a local mini-
PROBLEM 3. Let > 2 be an integer and a € (0, 00) such that mum point and f is unbounded on any set of the form /Q, where I a non-trivial
interval.
2+ logya n’ = ©) Let f : R — R, be function that is not bounded on any set of the form
10, for any non-degenerate interval I. Prove that f has not the property P.
Prove that N Claudiu Raicu
2logyn> a > 2logyn—
Radu Gologan 12" GRADE

PROBLEM 4. Let (P)3: an infinite family of planes and (X5, a family ProBLEM 1. Find all continuous functions f : R — R satisfving
of non-void sets of points, such that X, C P, and the orthogonal projection of
X1 0nto the plane P, is contained in X, for any n. A 1
Prove that there is a sequence of points (pu),31, such that p, € P, and p., v.“/ S@d=ns@)+ 3
is the orthogonal projection of pa1 onto the plane P, for any n.
Does the result remain valid if the sets X, are infinite? for any z € R and any positive integer n, n > 2.
Claudiu Raicu Mihai Piticari
12 PROPOSED PRoDLENS IMO AND BMO SevecTion Tests 13

PROBLEM 2. Let f € Z[X]. Forn € N, n > 2, define f : Zy — Zy, by


Fu(®@) = [(x), for any = € Z.
Find all polynomials f € Z[X], such that for any n € N, n > 2, the function
T is onto.
Bogdan Enescu
PRoOBLEM 3. Let £ :0,1] — R be an integrable function such that 1.3. THE NATIONAL MATHEMATICAL OLYMPIAD
1 Selection Tests for the BMO and IMO 2004
J@dz= [ af@)de=1.
Prove that
/ .FH)dz >4 First Selection Test
a
Ton Raga 1. Let a1,a2,a3,a4 be the lengths of the sides of a quadrilateral and s its
PROBLEM 4. Let K be a field of characteristics p, with p = 1 (mod4). semi-perimeter. Prove that
Prove that any non-zero clement in K can be written as the sum of three
squares of non-zero elements from K .
Marian Andronache

When does equality holds?


Glin Popeseu
2. Let R, i = 1,2,...,m, be a finite family of pairwise disjoint closed rect-
angular regions whose sides are parallel to the coordinate axes. It is also known
that the area of R = ) R, is at least 4, and the projection onto Oz of their union
is an interval.
Prove that R contains three points which are the vertices of a triangle of
area 1.
Dan Ismailescu
3. Find all injective functions £ : N — N such that for each n,

sy < 2@
Formulated by Cristinel Mortici
4. Consider an integer n > 2 and & disc D in the complex plane. Prove that
for any z1,25,..., 2 € D, there exists z € D such that 2% = 2,25+
220 2,
Barbu Berceanu, Dan Schwartz, Dan Marinescu
1 Proposep PronLexs IMO AXD BMO SELECTION TesTs 15
Second Selection Test 1. The incircle of the non-Jsosceles triangle ABC has center I and it touches
the sides BC,CA, AB, in ', B',C", respectively. The straight lines A4’ and BB’
5. A disc D is divided into 2n cqual sectors; n of them are colored in red intersect in P, AC and A'C' in M, and B'C" and BC in N.
and the other n are colored in blue. Starting from an arbitrarily chosen sector, we Prove that the straight lines P and MN are perpendicular.
count from 1 to n, in a clockwise order, the red sectors. We proceed in the same
way with the blue sectors, but in an counterclockwise order. Prove that there Classical result
exists a haltdisc of D which contains all the numbers from 1 to .
Kvant, M1684 12. Let n > 2 be an integer and a1,a, .., ax real numbers. Prove that for
any non-empty subset S C {1,2,...,n} the following inequality holds
6. Find nonnegative integers which can be reached by the expression
o +abt¥ (Za,)zg 3 (atetay)?
ah-1, e [
when a, b are nonnegative integers and ab # 1. Gabriel Dospineseu
Mircea Becheamu
7. Let a,b,c be integers, b odd, and consider the sequence zg = 4,z; = 0,
@3 =2¢,, 73 = 3b,
Fourth Selection Test
Tn = 0T+ Dns + ena, for n >4
Prove that if p is a prime and m a positive integer, then @, is divisible by p.
: Cilin Popescu 13. Let m, m > 2, be an integer. A positive integer n is called m-good if
8. A square ABCD is taken inside a circle . Inside the angle opposite to for every positive integer a, relatively prime to n, one has nfa™ ~ 1
£BAD one considers the circle tangent to the extended segments AB and AD and Show that any m-good number is at most dm(2™ — 1)
internally tangent to 7 at ;. The points B, C;, Dy are defined in the same way. Gabriel Dospinescu
Prove that the straight lines A4y, BB;, CCy, DD, are concurrent,
Radu Gologan
14. A point O is situated in the triangle’s ABC plane. A cirele C pass-
Third Selection Test ing through O is cut the second time by OA,0B,0C, in P,Q,R, respectively,
and C cuts the second time the circles (B,0,C), (4,0,C), (4,0, B) in K,L, M
9. Let n > 1 be a positive intoger and X be a set containing n elements. respectively.
Ay, Ay, .., Aoy are subsets of X such that the union of any 50 of them has more Prove that PK, QL, RM are concurrent.
than %n clements.
Prove that among the given subsets it is possible to choose three, such that
every two of them have a non-empty interscetion. 15. Some of the n faces of a polyhedron are colored in black in such a way
Gabriel Dospinescu that any two black faces have no vertex in common. All other faces are colored in
10. Prove that if n and m are integers, and m is odd, then white.
Prove that the number of edges that are common borders of two white faces,
1 (3m & is at loast .- 2.
Calin Popeseu
is an integer.
Cilin Popescu
16 Proposen Prouievis IMO D BMO SeLEcrion Tesrs 17
Fifth Selection Test 20. Given an integer number n 3 1, consider n distinct unit vectors in the
plane, which have a common origin at point 0. Suppose further that for some
non-negative integer m < n/2, on either side of any straight ine passing through
16. Consider a triangle ABC and O be an interior point ofit. The straight O, there are at loast m of these vectors. Prove that the length of the sum of all n
lines OA, OB, OC meet the sides of the triangle in A1, By, Cr, respectively. Let vectors cannot exceed n — 2m.
Ry, Ry, Ry be the radii of the circles (0, B,C),(0, C. ), (0, A, B) respectively Kvant
and R the radius of the cireuncirele of the triangle ABC. Prove that
04, 0B, 0cy
e o R
Dinu Serbiinescu

17. A move on am x n board consists of:


(i) Choosing some empty squares such that 1o two of them are in the ssme
vow or in the same column and placing a white stone on each of the selected
squares
(ii) Placing then a black stone on each empty square that corresponds to a
white stone on his row and on his colunn,
What is the maximum number of white stones which can appear on the
board, after some moves have been made?
2004 BMO Short-list, Serbia and Montenegro
18. Let p be an odd prime number, a;, i ,2,....p— 1 be Legendre’s
symbol of i relative to p (ie. a; = 1if =1/ 1 and a; = ~1 otherwise)
Consider the polynornial
f=ar+aX +o+ap X7 2
) Prove that 1 as a simple root of £ if and only if p = 3 (mod 4).
b) Prove that if p= 5 (mod 5), then 1 s a root of f of oxder exactly two.
Calin Popescu

Supplementary Test
19. Consider a sequence of positive different integers (@) such that there
is @ > 0 satisfying
an < an, for all positive integers n.
Prove that:
i) If @ < 5 the sequence contains infinitely many of numbers for which the
sum of their digits (in decimal representation) is not a multiple of 5.
i) The above result for a = 5.
Gabriel Dospinecu
JBAO SELECTION Tests 10
Second selection test

ProBLEN 5. Let ABC be an acute tringle and lot D be apoint on the


side BC. The points E and F are the projections of the poinit D on the sides
AB and AC. respectively. Lines BF and CFE meet at point P. Provethat 1D
1.4. THE NATIONAL MATHEMATICAL OLYMPIAD is the bisector fine of the angle BAC' if and ouly if the lines AP and BC are
perpendicnlar
Sele ion tests for the Junior BMO Severins Mokdoveann
PROBLEN 6. Au 8 x 8 array consists of 64 wnit squarcs. Inside el squave
are written the mmbers 1 or —1 so that in any 2 x 2 sub-aray the s of e
four numbers equals 2 or —2. Prove that there exist two rows in the arsay which
irst selection test the saume mnnbers are iuscribed in the same order.
Marins Ghong
PRORLEM 7. Consider a trinngle ABC with the side lengths a.h.c xo that «
PROBUEM 1. Find all positive real numbers a. b, which satisfy the inequa- is the greatest. Prove that ABC is a right triangle if and only if
lities
(Vai b+ va Bate+ a—o = (o+h+ev2
b+ be+ea) = 12?8 = 230" 5 ), Virgil Nienla
Laurentin Panaitopol ProBLEM 8. Find all positive integers n for which there are distinet integer
PROJ 2. Consider the numbers defined by a, = 3n + ViZ =1 aud wimbers ay. az, ..., ay such that
by = 2(VnZ 0+ Vi =), for all n = 1,2,....49. Prove that there arc integers Ln_mtakta
A, B so that a 2
Var b+ Vaz = by o+ Vg = b, = A+ BV Dinn Serbinesen
Titw Audreesen
ProsLen 3. Consider a circle of center O, and let, V be a point externally “Third selection test
to the circle. The tangents from V tovch the circle at points 1, T, Let T he &
point on the small are 71T, of the circle, The tangent at T interseets the line VT ProBuEM 9. Tn a chess tournament each of the players plages with all the
in the point A and the lines TT; and VT, intersect in the point /3. Let A1 be the others two games, one time with the white picces and then with the black picces
intersection point of the lines OM and AB. In each game the winner gets one point. and both players receive 0.5 points if the
Prove that. the lines OM and AB are perpendicular. gaane ends with a tie. At the end of the tonrmament, all the players end with the
Mircea Fiamt same munber of poins.
Protint 4. Consider a cube and let. M. N be two of its vertices. Assiign a) Prove that there are two players which liave the same number of fics
the mumber 1 fo these vertices and 0 to the other six vertices. We are allowed to b) Prove that there are two players which have the same nnmber of defeates
select a vertex and to increase with a unit the numbers assigned to the 3 adjacent when playing the white.
vertices and call this a movement. Prove that there is a sequence of movements Marins Ghergu
after that all the numbers assigned to all the vertices of the cube are equal, if and Pronven 10. Consider the triangle ABC, AB = AC and a variable point
only if MN is not a dingonal of a face of the cube. Al on the line BC, so that B is between M and C. Prove that the sum ofM. thei
Marius Ghergu, Dinu Serbitnescu in-radius of AMB and the ex-radius of AMC, corresponding to the augle
constant,
Virgil Nieula
JBMO SeLrcTion Trsts 21

2M 11 Let p.q,r be primes and let 1 be a positive integer such that defined by the conditions:
Prove that n = 1. ) on the first two rows, each element, starting with the third, is the suu of
the two preceding clements;
Laurentin Panaitopol ii) on the other rows, each element is the sum of the two cleny s placed
PROBLEN 12, Let 0 < b < ¢ < d ho positive intogerso that ad = he i above of the same column
Vil = i < 1. Prove that a is perfoet square. ) Prove that all the rows are defined according o condition i)
b) Consider 4 conscentive rows and lot a.b.c.d he the first clement in each
D Serbinesen of these rows, respectively. Find d in terms of a,b and c.
Dt Serbineson
Fourth selection test PROBLEN 18 . Let M, N. P be the midpoints of the sides BC.CA AT of
the triangle ABC. respectively, and let G be the controid of the triangle. Prove
that if BAIGP is cyclic and 2BN = V3AB. then triangle ABC is equilateral.
PROBLEM 13, Let ABC be a triangle inseribed in the cirele K. and consider BMO shordlist 2001
a point M on the are BC which does not contain A. The tangents from A7 1o
the in-circle of ABC intersect the circleK at the points N andP. Prove that if ProBLEM 19, Let A be n set of positive integers with the propertics
LBAC = LNMP, then the triangles ABC and MNP are congruent i) if a € A, then allpositive divisors of a ate elements of A:
Valentin Vornien ii)ifa.b€Aand 1 < a<b then 1+abe A Prove that if the set A hns at
least 3 clements, then 4 = N°
PROBLEM 14. The real numbers aj.as. ..., a, satisfy the equality Valentin Vornicn
afad o + (a1 aa o ) = 101 PrOBLEM 20. Consider a convex polygon with n > 5 sides. Prove that
Prove that lax| < 10. for all k = 12,100 there are at most ) triangles of area 1 whose vertices are choasen from
Ditni Serbinesen the vertices of the polygon
Andrei Nega
PROBLEN 15. A finite sot of positive integers is ealled isolated. if the sum
of the elements in any_proper subsct i a mher relatively prive with the sum
of the clements of the isolated set. Find all non-prite integers o for which there
exist positive integers a.b. 5o that the set A = {{a+ )%, (0 + 2b"..... (0 + b}
s iolated.
Gabriel Dospinescn
PrOBLEM 16. A regular polygon with 1000 sides has its vertices colored
in red, yollow or blue. A move consists in choosing to adjacent vertices colored
differenly and coloring them in the third color. Prove that there is a sequence of
maoves after which all the vertices of the polygon will have the same color
Marius Glerg

Fifth selection test

PropLEr 17, Consider the trinngnlar aray


01123 5 8
011 2 3 5
23’5 8
17
SuonTLISTED PrONLESS 2
Find the position of P for which the sum
PA+PB+PC+PD
has winimal value.
Mircea Becheam
1.5. THE NATIONAL MATHEMATICAL OLYMPIAD 7. Cousider a parallelogram ABCD such that m(£BCD) > 90° and BC >
€D 1t is known that for any points Af on DA and N on CB3 such that CN L D,
Shortlisted Problems for the Final Round oue has CAL = DN. Show that ABCD is a square.
Alexandrn Blaga
8. Cowsider a point. M on the diagonal BD of a given rectangle ABCD,
7" and 8" GRADE el that ZAMC = ZCMD. The point. N is the intersection point betsen AM
and the paralicl e to CAZ that contains B
1. Paint in red n of the vertices of a regular octagon. v < 8. Find, with Prove that the triangle BATN is cquilateral if and only it ABCD is a squuare.
proof, the smallest value of n which insures the existence of an isosceles triangle Valentin Voruien
having only red vertices. 9. Given a set X containing n elements (n > 2). find the mumber of pairs
Radu Gologan (4.B) such that AC BC X, B% X and A has more that one element.”
Valentin Voruien
2. Let a,b,c be non- ero integers such that @ > 0. be > a and ac + b > 3
Prove that ab+c > 2a.
Lucian Dragomir 10. For n.p positive integers. consider f(n,p) = [T/
) Prove that f(n.3)+ f(n+ 1.3) + [(n +2.3) i a perfect square.
. For a positive infeger ot ending with0, we define it reverse10 he the b) Prove that i f(n) = Fn.p) -+ F(n-+ Lp) + f(n +2.p) is a perfect square
wamber obained by writing the given mumber in reverse order (for example, the for any positive integer n. then p
reverse of 1234 is 4321). Find all positive integers n such that Marins Burten
) can e written as the difference hetween a three digit unmber and its
reverse:
b) n? can o witten as the difference hotween a four-is muber and its 11, Find all functions [ : R — B. having the property that
f20=5) < 20 =3 < f(20) =5,
Valeutin Vornien
for any real 7.
1. Tet a.b be positive integers such that Vi + Lf s an inteser. Prove that Liliana Autonesen
o and b are perfoct squares, v
Mircen Becheann
9'" GRADE
5. Consider an ncute triangle ABC of orthocenter H and altitudes AM.
BN,CP. Denoteby Q and R the midpoints of BH and CI1. vespectively. Consider
the following " intersections [ QAR V = MROAC.T = AHA P) 12. The positive mumbers a.b are given such that a + b = 1. Find the
Prove that T is the orthocenter of the triangle UAV. al value of
Manela Prajea
6. Let P be a variable point on the border of a rectangle ABCD, where losn V. Maltei
AB=CD=u, BC=AD=band a > .
24 Provosep tromExs SHoRTLSTED PROBLEMS 25
13. The positive numbers a, b, satisfy abe = 1. Prove that 20. Find all pairs (.y) of real numbers, that satisfy
T a(Sarytoa). Haat
oy = Vi ent e
Cabriel Dospitiescn Titn Audreescn
14. Let a,b € R and let / : R — R, be an increasing function. Find the 21. Prove that for real mumbersa, b, e such that a2 +522 9. the following
functions g = [a.b] — [f(a), (b)) satisfying g(z) ~ g(y) = |f(x) = f(y)]. for any inequality holds
.y € [a,b] 3minfa.b.c} < 1+ abe.
Cilin Burdusel
Virgil Nieula
15. Find the quadratic functions f(x) = w4 b+ e, with [(0) € 2 having
the property that 22. Let P be a positive mumber,g € {57+ 1,7 +2.....p% + 2} and i, =
[(p+ /@)"], for any non-negative integer .
P R L B Prove that there is a one-to-one correspondence between the sets

{neN|z, iseven} and {n€N|risodd).


hold for infinitely many integer values of n.
Cristinel Mortici Radu Miculescn
16. For any integer n, n > 2, put an =2+ 4= + L 4.4 L 3. Find all positive integers n, such that the following statement holds: for
AR 7
8

any non-zero real numbers a,b, there are real numbors 1.7z, ..., for which
Prove that 1+ 2 Rk n <,
Cristinel Mortici franin
17. Given positive mumbers a,b, ¢, prove the incquality EREAS
Yoo, Mihai Baluna
clc+a) ~ Bathre? 24. Cousider a triangle ABC and a real mumber k. k > 1. Let A B'.C" be
Petre Batrinetn points on the sides (BC), (AC), (AB) respectively, such that
18. Find all quadratic functions (z) = az? + be + e, where a.b.c arc real 1_BA i_cB
mumbers and a # 0, having the property that the images of the intervals [0, 1] and ey kS AR <k
[4.5] are two intervals having exactly one common point. their union being the Prove that,
interval [1,9]. max(A'B, ACBC) K
Cristinel Mortici max(AB, AC,BC) - k+ 1
10. For positive numbers a,b,¢, prove an Tsmailescu
3" Va + ) = 2vabela? + b+ ), 25. Prove that a triangle ABC is equilateral if and only if there are points
M., P on sides (AB), (BC), (CA) respectively, such that the triangles ABC and
Marin Chirein
AINP iave the same centroid and the same orthocenter
Marian Tonesen
2 Prarostn enou s SHORTLISTED PROBLEMS 2
26. Prove 33. Let M be an interior point in the tetracdon ABCD and let A'. B'.C", D
be the intersections of the lines AM. BM.CAL DA with the opposite phines,
1 N 3 N 9 respectively. Prove that )
l97
= 3tan9’ | cot27e - Fum2r | corsle ~ Bansle S AMYo 20
tan 27 = 101an9°.
Andei Clites
Titw Awdrecsen
27. Tn the convex quadrilteral ABCD suppose that AD = DB, AC = €D
and { BAC = 2LBDC.
Prove that (CBD = 30° and that there is.r € (0.30°) such that G0+, 11" and 12" GRADES
LB =90+z, LC =150~
Virgil Nicula
3. Let ()31 be u sequence of real numbers greater than 1. and sippose
28. Prove that & quadrilateral ABCD is a rhombus if and ouly if for any. that a1 > 2a, — 1 for any positive integer n
point M, in its plane, the following inequality holds a) Prove that the sequence (z,),1. defined by
AMA-MC+ MB-MD) 3 AC* + BD* o\ 1 V¥n21,
aay-ay
Lawrentin Panaitopol
has a finite it .
29. Let a € (0.00)
\ {1}. Find the real munbers .. = that satisfy b) The sequence {a, )1 i defined by
ot log,
a¥ +log, an(ansy =1)"
o +log, s =a. for all positiven. Prove that (a,.), is monotonic and find its limit
Marian Tetiva

30. Find all pairs (z1,22) of complex numbers, satisfying the following two 35. Find the set of all real numbers x, such that there is an increasing
conditions: seqenco consisting of positive integers (1,31 snch that. the seence (),>1
(a) 1+ 1142 defined by
(b) 212221 + 2] 2J21] + z2)). (20 499 40 +2)
Valentin Von
31. Let 7,5,be positive mumbers, such that £+ + 2 = 5. Prove that satisfies lim b, = .
Radu Gologan
(1= sinz)(1 = sing)(1 — sinz) > sinzsiny sin = defined by 41 = n + 7, where
36. The sequence ()31 s recurrently
Find the cases when the equality holds. e Prove that:
Gheorghe S7lisy ) The scquance defined by y, = % Ina, > 1, is bonnded and monotone;
32, Prove that a tetrahedron whose edges are in arithmetic progression and b) There is a positive real number A such that. lim )“'T“ 1
the 3 pairs of opposite edges are perpendicular, is regular. Cristinel Mortici
Mireea Becheann
28 Prorosen proness SnorISTED PrORENS 2
37. Letn > 1 be an integer. Prove that for any 7 € by there is a . Let g R B be such that g is ncreasing and. Jin F) = af(0) for
sequence (ax(r)) 31 sich that ay(x) € {0.1}, k € N, and any
Prove that f is contimous,

o [ o « briel Dospinesen
Afx(muy EreT ’mu-w) 45. Find all functions J : (0,00)
— R having the following propertis:
a) f(x) +2r = f(3x) for any x> 0
Marian Tetivs Gabriel Dospinesen b) Jim (/) 1) =
38, Prove that for any i« € [0.50) there are soquences of positive intesers abricl Dospinescu
(@0 and (b s that
46. Lot f T — I, where I C Riis an interval. be a contimons fimction snel
) that for any € T there i n, € N* such that [ () = r.
o (G
Tin —_——
tn + b
) IF 1 = [a,0). prove that f(x) = . for any = € I
1) Prove that if f s not the identity fiunction in the case = (). then
lim ()= a.
39. Prove that the sequence defined by , = log, nl. for any pasitive integer Dan Selwarz
1, doés not contain infinite arithmetic sets.
Adrian Tuga A7 Let, A, B be n by n matrices such there are non-zera real P
with the properties 4% = KBS and pAB + qBA = 1, Prove that if A3 4 BA
40. Given the non-negative integers, when is the sequence defined by then =g
Valentin Vornien
abz0
48. For a given positive n, one considers the matrix
convergent?
Dan
41 Let (x,), be asequence of real mumbers such that the sequences g, =
Tty 0 Z, = oyt 7, are convergent.
Prove that (z,), is convergent,
Mibai Pitieari Prove that A is now-singular and the sum of the clements in A= i 1
42. Let k be a positive integer. Prove that if a sequence (a,,), satisfies Toan Rasa. Mircea Dan Rus
(] = fan]* + Ok + D] + 1. 49. Find all two by two matrices A, having integer elements, svich that
for any positive n has a finite limit, then k = 2, det(A° + 1) =
Cilin Popescu, Sergiu Romagen
Mireea Becheann
43 Let fo :[0,1] — R, be a sequence of functions such that | £, ()~
|z~ yl for any 2, € [0,1] and any positive integer n. Let f : [0,1] = f,R ()]
be <a 50. Let A € My(Z) be a non-zero matrix. Prove that the following proper-
continuous function such that for any rational musnber,= & [0.1], we hove ties ave cquivalent:
i fu) = £(2), ) 4det A = ()% 2 and Ais. not of the foru (1),
k0 with & u integer;
b) For any matrix B & My(Z) commuting with A, the mumber det(A? + 52)
Prove that i fo(@) = f(z) for any = € [0.1]. s the square of an integer.
Radu Gologan Gabriel Dospinescn
30 [T —— SHOMTLSTED PrORLES 31

51 Leta > 0 and a > 0. Find all intezrable functions £ : [0.a] — [0. %), . B6. Let:[0.1] = R be a fanction which has continons derivative such that
such that for any « € [0,0] 1f()] < 1. for all x € [0.1]. Denote by f, = [o fo-++o f the n-th composition
of f with itself.
#) Prove that. the sequence 1, = [ fi (r) die, has finite liniit
s [Cerma b) Let I be the limit defined ahove and (.,),, the sequence defined by

Calin Popesen. Valentin Voruicn


JRZER
Let ()30 be avsequence with the property | Prove that (J,), s a fiite Jimit
Nelu Cliichirim

Dan §tefan Marineseu, Viorel Corea

53, Consider the continuous functions f1. f 0.1] — R that satisfy


the following conditions:
'
a) /(/k(r];ldaslfnrk 2
by }‘x(l’) + fa(x) + -+ fulz) = 0, for all x € 0.1].
Prove the incquality

g(/«'un ) s
Calin Popescu
54. Onside BC. of the triangle ABC. one considers a variable point P such
that ZAPB =t and PB = . Prove that
1mer
/ cost(z)dr = AB - AC.
o
55. Let 1 C (0,00) be an interval and g : T — R an integrable funetion,
Prove that there are real mumbers .y such that
ety
%/ sty < 1.
Cristinel Mortici
3 Prorosn Prowis REGoxAL Courrrmons 33
2. Let 2., be real numbers such that
2 b€ P <2 Py <2,
Find the minimal and the maximal value of the s v+ + -
2001 Foesani Contost
1.6. REGIONAL MATHEMATICAL COMPETITIONS
. We are given the set A = {1.3.6.10.15.21..}. Show that there exist
Selected Problems IS a1 . g1 € A such that
@+ 02 @+ s = azor
2001, Focsani. Bogdan Euesen
7" GRADE
. The circles €y and Cy interseet in distinet. poiuts 4. 5. A arbitrary
1. Let . be positive integers. Show that 25% — 7% is divisible by 3 and throngh A intersects again Cy in Cand Cy in D and let AT be st arbitrary point liveo
find the least positive integer of the form (25" — 7 — 37|, when . vun over the. the segmeut. CD. The parallel live to BC through Al intersects the segment 5L
set of non-negative integors. in K and the parallel to BD through Al intersects the segment BC i N. The
2001, tasi. Marins Ghergn perpondicilar in N to BC intersects the arc BC of €, which does 1ot contain A
in the point B. The perpendicular to BD in K intersects the arc BD ofC; which
2. Let a.h be seal mambers such that fa > 2. [b] > 2. Show that does not contain A in F.
Show that. ZEMF = 00°.
(0 + 1 +1) 2 (a+b)ab+
1) + 5. 2004, Foexani Contest
5. Let n be a positive integor and a, b, be real ambers such that a” = a-+h,
e find when equality holds. B=btcandc" =cta
201, usi Show that a
2001, Tassy Contest
3. Let ABC be a triangle. M be the foot of the altitnde from € and N e
the reflection of M across the Tine BC. The parallel linc to CAL through the point 6. Let ABCD be a convex quadsilateral aud AN, P.Q be points on the
N interseets BC in P and AC in Q. sides AB, BC.CD. DA respectively. such that
) Show that MQ L AP if and only if AB = AC,
1) Shaw that it is possible to obtain the points A, . * when the points MA_NB_PD_QA
AN, P are given. BT NCTFC T QD
2001, Tasi
where k % 1. Show that S(ABCD) = 25N PQ) if and only if S(ABD) =
S(BCD)
2004, Tassy, Potre Astafei
9" GRADE
7. Let ABC be a vight triangle such that, LA = 90°, /B > £C. and Jet
1. Let ab,c be veal mumbers. Show that D be an arbitrary point on the scgment. BC. The angle bisectors of ZADE and
LADC intersect the sides AB and AC in the points M and N respectively. Show
that the angle between the lines BC and AN is (B - C) if and anly if D is the
Vit o+ v VaThTe foot,of the altitude from A
2004, Tassy, Bogdan Encsen
if and only if
S b et = (a+ b+ o) 8. Fiud all real numbers 7, 7 > 1, seh that 3/[7] is an intoger number for
all positive iutegers . n > 2.
2004, Focsani Contest, Bogdan Encsen 2004, Tassy, Mibai Piticari
34 Proross Proms RecionaL Courrrimions 35

10" GRADE Show that if AQ LAB and AQ; = }AD, then the point Q does not lie
inside the triangle AP Py
1. Find all arithmetic sequences
n. na, na. 1.0 for which 5. 2fua. 113, 2004, Foesani. Mircea Bechenm
T and 17)s 2. Lot A,N be 2 x 2 real matrices such that AN VA and X 0 for
2004, Focsani Contest
some positive integor m. Show that
2. Let ABCD e a convex quadrilateral and A7, N. P, be the widpointsof ) det(A+ N) = det A
ihe sides AB. BC, CD. DA respectively. b) For det A # 0, 4+ is invertible and (A + M)~ = (A= N)A~2
Show that if ANP and CMQ ave equilateral triangles then ABCD is a 2001, Foesai. Ton D. T
shombus. Find the angles of ABCD.
2001, Foesni 3. a) Prove that for all positive intogors . the followi incq ity holds
| Lot A = {1.2.3,4.5). Find the munber of functions f : 4 — A, with
the following property: there i 1o fripleof distinet clements a, b.r & A such that
Ja) = [(b)
= f(e)
2004, Tassy, Adrian Zanosehi
(2
b) Show that the sequence of real numbers (a,), 31 defiued by ) = 1 and
4. Let a2 2 he a an integer. Consider the et
A={Va.
Va Va.
S b forall > 1
&) Show that A does not contain an infinite geometric ratio. is monotonic inereasing. Find with proof if it is a convergent sequerice
b) Slhow that for suy n > 3. A contains 1 wumbers which are i a zeometric 2004.Forsni
ratio.
2001, Tassy. Bocdan Enescn 1. Let @ € (0.1) be a real number and f ¢ B — R he n function which
satisfies the conditions
5. Let ABCD be a tetrahedron such that the medians starting from vortex
Ain the triangles ABC. ABD, ACD are mmtually perpendicular. Show that all
edges that contain A are equal.
2004, Tasy. it Serbanensen ) i PL= S00)
6. Let .y, be real numbers such that Show that. lim 120
2004, Focsani. Mircen Bechieam
o8z + cosy + o5z =0
0331+ cos 3y + cos 3 5. Let ABCD e a paraliclogram of nequal sides. The point I is the foot
of the perpendicular from B 1o AC. The Tine through E which i perpendicular
Prove that c0s2z cos2ycos2 <0 to BD intersects BC in F and AD in G.
2004, Tussy, Bogdan Enesen Show that EF = EG if and ouly ABOD is a rectangle.
2004 Lassy, Mireon Bechesnm
6. Let A.B be 2:by-2 matrices with integer entries, such (hat AB = B.1
11" GRADE o ot B = 1
Prove that if det(A®
+ B%) = 1 then 42 = I
1. We are given a rectangle ABCD and let P be an arbitrary point on the 2004, Tassy. Mircen Bocheann
diagonal BD, P # B,P # D, and Q be an arbitrary point insice the tringle
ABD. The perpendicular projections of P on the sides A5, AD are Py. P xespec-
tively, and the perpendicular projections of @ on the sides AB,AD are Q. Qe
respectively.
36 Puoroseh Prosas
12" GRADE
1. Let G be a gronp such that every clement . r # 1. hins order p.
@) Show that p is a prime mumber
D) Show that if any p? — 1 clement subset of G contains p elements which
commute one to another. then G is an Ahelian gronp
2004, Tssy. Clandin Raien
2.1. THE NATIONAL MATHEMATICAL OLYMPIAD
District round - Solutions

7" GRADE

PRoBLEM 1. Find the number of positive 6 digit integers such that the sum
of their digits is 9, and four of ts digits are 1,0,0,4.
Solution. The pair of missing digits must be 1,2 or 0,3.
In the first case the first digit can be 1,2 or 4. When 1 is the first digt, the
remaining digits, (1,2,0,0,4), can be arrangod in 60 ways. When 4 or 2 s the first
digit, the remaining ones can be arranged in 30 ways.
In the same way, when completing with the pair (0,3), the first digit can be
1,3 or 4. In cach case, the remaining ones (three zeros and two distinct non-zero
digits) can be arranged in 20 ways.
In conclusion, we have 60 +2 30 +3 20 = 180 numbers which satisfy the
given property.
PROBLEM 2. Let D be a point on side BC of a given triangle ABC. The
bisector lines of the angles ZADB and £ADC intersect AB and AC at M and N,
respectively, and the bisector lincs of the angles ZABD and LACD intersect DM
and DN at K and L, respectively.
Prove that AM = AN if and only if MN and KL are parallel
Solution. In the triangle ABD, K is the incenter, so AK is the bisector
line of ¢BAD. Tn the same way in the triangle ADC, AL is the bisector line of
LDAC.
Using the bisector theorem in triangles AMD and ANC (for the lines AK
and AL), we got:
AM MK g AN NL
4D ~ KD " AD T ID
But AM = AN if and only it 25
KD~
= YL
LD
it o only if KLIMN (by Thales
theorem).
40 Prorosep PROBLEMS AND SOLUTION
SoLUTIONS - DiSTRICT RoUND a
PROBLEM 3. One considers the set A = {n eN 1< \i+va< 2}. a common edge, we get APQB = ANQB. Tt follows LPQB = (NQB and, as
a) Describe the set A. LCQA = LNQB, we obtain ZAQC = LPQB.
b) Find the set of numbers n € A such that o b) Because of the similarity of the triangles ADQ and RQC, we have
oD gq_ 1t follows that the triangles DRQ and ACQ are similar, too. This
a 1+ vA| <17 gives that the corresponding angles are equal, thus ZDRQ = 45°.

Solution. a) We have 1 + v/ > 2, that is v/1+ v > 1 8% GRADE


The inequality v/T+ v/ < 2 is equivalent to 1+ v < 4, or n < 9. We gt
A={12,....8 PROBLEM 1. We say that the real numbers
a §i b have propertyP ifa®+b € @
b) We have [1 = /T Va| = /T vV 1. The inequality /15 Vi~ 1 < and b +a € Q. Prove that:
1+v2 sib= —;—fi are irrational and satisfy P;
is cquivalentto y/TF
v/ < 1+Vm Ve Lori< o+
, that is 1< 1,1 &) The numbers a
For n > 3 we have b) Ifa, b have property P and a-+b € Q\{1}, then a si bare rational numbers;
©) I o,b have property P and § € Q, then a si b are rational mumbers.
It follows that the given inequality is true if and only if n € {1,2}. Solution.
. a) We have a? = fl g 32
o that
is o 4 b =
ProBLEM 4. In the triangle ABC we have AB = AC. Consider the points atB=7eQ
M, P on AB such that AM = BP. Let D) be the midpoint of BC and let R on CM b) From a?+b— (1 +a) € Q we get (6~ b)(a+b~1)€ Q. Because a+b—1
and Q on BC, be such that 4, R, Q are collinear and the line AQ is perpendicular is a non-zero rational number, we have a—b € Q. As a+b € Q, we get 20,20 € Q,
on CM. that is a,b € Q.
Prove that:
) Let k € @\ {0,1} such £y that a = bk. We infer b(1 + k%) € Q and
b(b+k) € Q, implying % —reQ Ifr=1k then k= . This implies
Solution. We suppose without loss of generality that the order of points is
AM,P.B. 3 and a = b, a contradiction. Thus r # k2, from where b
result a = & beQ.

PROBLEM 2. The real numbers a,b,¢,d satisfy a > b > ¢ > d and
a+bte+d=2004 si a®— b+
- =2004.
Answer, with proof, to the following questions:
) What is the smallest possible value of a?
b) What s the number of possible values of a?
Solution. a) One can wiite a? — 12 + ¢ — & = (a —b)(a +b) + (¢ —
d)c+d) > a+btctd=2004 la—b>1orc—d > 1, we casily deduce
Let I be such that ABEC is a square having center D and N is the inter- @2+—d? > 2004. Thusa—b=1c—d=1, thatis b=a—Ld=c~1,
section point of AQ and BE. case in which o+ + ¢ +d = 20+ 2 — 2 = 2004, This implies a + ¢ = 1003. As
) As the triangles AMC and NBA are equal, we get AM = BN. As a> ¢, we get a > 502. For a = 503, we obtain b= 501,¢ = 501,d = 500, thus
AM = PB, we obtain PB = BN. Because LABC = (CBE and the line QB is b=c, which does not satisfy the given condition.
For a = 503 one obtains b = 502, = 500,d = 499.
a2 Proosep PROBLEMS AND SOLUTION SoLuTIoNs - DisTRICT Rouxp 43
b) The maximal value ofa is obtained for the smallest value of d. When d b) We shall use the notations AB = b and BC = c. Because the triangle
we got¢ =2, 6 = 1001. Thus a € {503,504, ..., 1001}, taking 1001 - 503-+1 = 499 MNP is equilateral we have
possible values. To check that any o from the se. {503,504,..., 1001} gives a
solution, takeb = a—1, ¢ = 1003 —a,d = 102~ a. It is easy to sce that
L
o .
a>b>c>d, and the cqualities from the hypothesis are fulflled.
PROBLEM 3. Say that a set of three different numbers s an arithmetical and consequently a =
set if one of the three numbers is the average of the other two. Consider the set 10 the midpornt of the sogment MY, then PO L (DMN). Tt follows
An={1,2,...,n}, where n is a positive integer, n > 3. that the planes (PNM) and (DMN) are perpendicular. If DF L NM, F € NM,
a) How many arithmetical sets are in 417 then DF L (MNP). Thus
b) Find the smallest n, such that the number of arithmetical sets in A, is
greater than 2004, DN-DM _ av3
Solution. a) It easy to see, by inspection, that there are 20 arithmetical PP==mr =73
subsets of A
b) Consider an arithmetical set B = {a,a + r,a+ 2r}. For each r < 45 and
1<a<91-2r, we have B C An. As a consequence, for any r € {1,2,...,45} 9" GRADE
there are at loast 91 — 2r such subsots of Ao;. The total number of theso subsets
i5 14345+ +89 = 2025. In the same way we can see that the number of
arithmetical subsets in Aoo is smaller than 2000. PROBLEM 1. Real numbers a, b,c satisfy a2 +? + ¢ = 3. Prove the inequa-
ProBLEM 4. Tn a given trapezoid ABCD, let AB | CD, /B = 90° and lity:
AB = 2DC. Consider points N and P on the same part of the plane (ABC), such [a] + 1b] + c| — abe < .
that PA and ND are perpendicular on the trapezoid plane, and ND = a, AP = 2 Solution. The Cauchy-Schwarz inequality yiclds
(@ > 0). If M is the midpoint of BC and the triangle MNP is equilateral, find:
a) The cosines of the angle between the planes (MNP) and (ABC);
b) The distance from D to the plane (MNP). (ol+ 1+ e)? <3(a?
42 +.)
hence
Solution. lal + 1]+ e < 3.
From the AM-GM inequality it follows that
@412+ > 33/ (abo,
or Jabe| < 1, which implies —abe < 1. The requested inequality s then obtained
by summation.
PROBLEM 2. Find the cartesian coordinates of the vertices A,B,C of a
triangle ABC, whose the orthocenter is H(~3, 10), the circumcenter is O(~2, ~3),
and the midpoint of BC is D(1,3)
Solution. We have AH = 20D, implying that the coordinates of the point
) The orthogonal projection of the triangle PMN onto the plane (ABC)is Aare (~0,-2) and the radius of the circumcircle equals VA The line BC is
the triangle AMD. By the projection theorem
perpendicular on OD, and its equation is z + 2y = 7. Since BO = CO = /50, we
deduce that the coordinates of the points B, C' are (3,2) and (~1,4)
PROBLEM 3. a) Prove that there are infinitely many rational positive num-
bers x such that:
where o is the angle between planes. {2} +{z} = 0,90.
a4 Prorosep PRoBLEMS AND SOLUTION SoLutions - DisTRicT Rouxp 5
b) Prove that there are no rational numbers > 0 such that; R :
For the sum to cqual G, the manbers 3 (s ~
+{=} and L (vp — tp) imu -2 E p have to be even. This happens only if
Solution. a) Since 0.99 = 25 it is natural to look for a rational z of the he et M\ {4] contain an even nutaber of points hving on odd abscisa and an
even number of points having an odd ordinate, which is not possible.
form 5, for some positive integer n. It is not dificult to see that 7 = 1 satisfies
the given equality and then that = 10k + & also satisfies the equality for any
positive intager . 10" GRADE
b) Suppose, by way of contradiction, that such a rational number exists.
Letn= [z]; then
n < = <n+1, hencen® < 2% < n® +2n+ 1. Let [z?] 7n’+k ProBLEM 1. Given a positive integer n, 7 > 3, find the number of arithme-
where 0 < k < 2n. Substituting {z} = = — [z} = © —n and {z?} tical progressions with 3 elements contained in the set, {1,2,....,n}
2~ ~ I in the given equality, yields
Solution. Let us count the subsets according to the value of the common
2 tz-n?en-k-1 difference of the progressions. There are n—2 progressions with the connmnon differ-
ence equal to L {1,2,3},{2,3,4},....{n—2,n~1,n). There ate
Consider the above equality as a quadratic equation for the unknown . Since & n— 4 progressions with the commaon diference equal to 2: {1,3,5), (2,46 ,.
‘must be rational, its discriminant A = 4n? + 4n + 4k +5 has to be an odd square, {n—4,n—2,n).If n is even, say n = 2k, the maximal value of the common di
say (2m ¢ 1)%. We deduce that 1 4 +k + 1= m? + m. Clearly, m > n+1, and foronca is - 1; there are two such progressions: {1,,2% ~ 1} and {2,k + 1,26}
hence m? +m > n? + 3n-+ 2. Finally, we obtain k > 2n + 1, a contradiction. Thus, in this case, the total number of subsets is 2-+4+ -+ + 2k — 2 = k (k — 1)
Tf i is odd, say n = 2k + 1, the maximal value of the common difference %i
J and there is only one such progression: {1,k + 1,2k + 1} . The totel number of
PROBLEM 4. A rectangle 2 x 4 Is divided in § squares of side 1. Call A the subsetsis 1+ 3+ ...+ (2k— 1) = K2
set of the 15 vertices thus obtained.
Find the points A € M satisfying the following condition: the set M\ {A} T both cases, the number of subsets equals [3] [L"]
can be arranged in pairs (A1, B1), (A2, Ba), ..., (Az, By) such that
PROBLEM 2. Find integersn, n > 3, having the property: there are distinct
ABl+ LB+ + AiBr =T, integers a1, az, .., such that:
Solution. We will prove that the poin A s either the center of the rectangle aslay!
or the midpoint of one of its smaller sides. In these cases, the requested pairs can
be chosen as in the figure. Solution. We prove inductively that for all n > 3 such numbers exist
Indeed, for n = 3 we have 3! 5! = 6. Assume that for some n there exist a; <
B As A, B: B A B B A Bs a3 <+ < a, such that
Al o2294;B ol ap!
A, B, B, By A, A By B B 4 !~ 1. Then a, < band
For other positions of the point A, consider M — {0,1,2,3,4) {0,1,2} arlagle gl
Suppose the pactiton into paiss possbl, and donote A (& ) By (1) o Thus, denoting b = Gy, an! = @41, we have
p=12....7. Than
@bl il ol = Gl
as desired.
46 ProvosED PROBLEMS AND SOLUTION SoLutions - DisTriCT RounD a7
PROBLLEM 3. Let M, N, P, and Q, be the midpoints of the edges AB,CD, a contradiction. A similar contradiction is reached by assuming z > . It follows
AC, and BD, respectively, of the thetrahedron ABCD. It is known that MN is that 2 = y, which implies the conclusion.
perpendicular to both AB and CD and PQ is perpendicular to both AC and BD.
Prove that AB = CD, BC = DA, and AC = BD,
11" GRADE
Solution. DenoteAB = z, AC = y, andAD = =, Thon M = —2-V+%
and since 3N - 4B = MN - 0D = 0, we obtain PROBLEM 1. Let 2o > 0 and for any positive integer n, consider Zn41 =
Znt—=
zzay -2t =0 Ve 2
¥ - —ay+zz=0, Tind: ) lm z; b) lm 7%
Solution. a) It is not difficult o see that (zn),5o Is strictly increasing,
Substracting these equalities yields z2 = (z — y)?, hence AD = BC. Adding them 1
up yields y* = (z —y)?, thus AC = B. hence lim, = L exists. If L € R, then the recursive relation gives L+—,
+7E
In the same way, the second condition of perpendicularity leads to AB = CD. which is impossible. Tt follows that I = co.
Alternative solution. It easy to soe that MN is a median and an altitude b) First, observe that
in the triangle ABN; thus the triangle s isosceles. This implies BN = AN. In Tnt1
the same way one can prove that AQ = CQ, MC = MD and BQ = QD. Using
the formula for the median line, we easily get the equalities of pairs of edges.
PROBLEM 4. Let 7,y € (0,5). Prove that if the equality from which we deduce that
lim
ooz,
(cosz iny)" = cosnz + isinny 5
Tn | using the Stolz lemma:
is true for two consecutive integers, then it is true for all integers n.
Solution. Suppose
(cosz +isiny)" = cosnz
+ isinny
(cos +isiny)™* = cos(n+ 1)z +isin(n+1)y.
Multiplying the first relation by cosz +isinz and substracting the second, we get
sinzsinne = sinysinny.
Suppose z < y. Then sinz < siny, and hence [sinnz| > [sinny| PROBLEM 2. Consider complex non-zero numbers 21,2, ..,72, 3 3,
such that [z1] = |z = -+ = |2n43] and argz; > - > arg(za43). Define for
Considering the absolute values in the equality (cosz +ising)" = cosnz + ij € {L2nk by = |z = Zan| and let B = (by) € M. Prove that
isinny, we obtain det B =0,
cos x4 sin?y)" = cos?nz + sin . Solution. Consider = (cos2c +isin2z) and w = r (cos2y +isin2y),
then |z — w| = 2r |sin (z — y)| . Using the hypothesis we can reduce detB to the
Thus, we have form
sin(z)
— Tn41) sin(z) —Taga) sin(@)— Tnys)
(cos? z+ sin? 2)" < (cos? z + sin?y)" sin(zy — Tntr) Sin(Tn — Tng2) SI(Tn — Tnss)
cosna 4 sin? ny)” < cosPnz + sin?nz = 1, Writing it as a sum of determinants, we reach the desired conclusion.
48 ProPOSED PROBLEMS AND SOLUTION Sowutions - DisTricT Rouxp 49
Pronuex 3. Let f : R — B be a function with the property P: for every Solution. ) rank A = n implies A invertible. As rank AB < rank(B) =
a,be R e have f (%b) € {f(a), FB)}. and rank AB > rank(A~1AB) = rank B = r, we get rank(AB) = 1.
b) Consider A € Sy, and the function f : S, — S,, f(X) = AX. It easy to
a) Give an example of & non-constant function possesing P. see that; A is one-to-one (and thus onto) if and only if is one-to-one and onto
b) If £ satisfies P and is continuous, prove that it is constant. (the set S, isfnite).
Solution. a) An example is the fnction Wethusget § = 57 X = 30 AX = AS, which can he written (A—1)S =
xes. " yes,
0 z<0 In order to prove S 0, it suffices to show that there is an A,,€ Sy, such
f@={1 51
that A, — I is invertible. Take for example, for n =2 and n = 3 Ay = % H
b) Observe that if ¢ < d and 7 (c)= f (d) , then f is constant on the interval 001
fe,d]. Indeed, let my be the midpoint of [lfl .q let mz and s be the midpoints Ag={1 1 0). In general consider matrices of the form
of [e,m] and [m1,d] and so forth. The set of these points is dense in [c,d], and 010
since / is continuous, the conclusion follows.
Now, suppose there exist o < b such that f(a) # f(b), and let & =
sup{z € [a,b] | f (a)= f (¥)} and b, = inf { & [a,b] | f (b) = f (z)}. Then
f (a) = 4 0 0
T{a1), £ () = ] (bs), hence J is constant on both intervals [a,as] and [by,1] . But 0
Agn = s A=
f(“l +‘7') ¢ (f (1), f (b)), & contradiction. Ay

PROBLEM 4. The matrix A = (a;;) & M,(C) is defined by ais = azs = ProBLEM 2. Prove that constant functions are the only continuous functions
<= ap-1,p =1 end ay; = 0 for the remaining set of indices (i, ). £:10,1] = B, such that
Prove that there are no non-zero matrices B,C € My(C) such that
(Ip + A)" = B* + C" for all non-negative integer n.
Solution. From (I, + A)* = B2 + C? and (I, + A)° = B? + C? we obtain [ t@n@as= [ swrae- [ ooz,
BC =CB =0,. The equality I, + A= B+ C implies B + AB = B? + BC = B?
and also B + BA = B? 4+ CB = B*. Hence AB = BA and, similarily, AC' A for any non-derivable continuous function g [0, 1]
Solution. I f verifies the given property, then, for every real ¢, the relation

el )
It is easy to see that it follows that there exist b, ¢ such that
is also verified by f — ¢. Thus, the function

h(x) = fa) -
with b+ ¢ = 1. The equality BC = 0, implies be = 0, so either b or ¢ equals 0. If, verifies the equality
for instance, b = 0, then it follows that ¢ = 1, hence C is invertible. In this case,
the equality BC = 0, implies B = 0, as desired.
.
@ [ (@)dz =0, for all g continuous and non-derivable.
12" GRADE
PROBLEM 1. Let n > 2 be an integer and r € {1,2,...,n}. Consider the set If, by contradiction, h is not the zero function, take zo € (0, 1), such that h(zo) £ 0.
Consider an interval of the form V = [0 —¢, zo+] C (0, 1) where & > 0, on which
= {A € Ma(Z) | rank A = r} i has constant sign. Such an interval exists by the intermediate value property.
‘The function defined on [0, 1] by 9(z) = (z ~20)* —* for = € V and g(x) = 0
a) Prove that for any A € S, and B € ,, AB is in Sy; otherwise, is not derivable. The integral i (1) is not null, a contradiction.
b) Caleulate Y~ X. PROBLEM 3. A ring A satisfyies the following properties:
Aes. i) Its unit 14 has order p, & prime number;
50 Prorosen PRoBLEMS AND SOLUTION
i) There is a set B C A with p elements such that: for any z,y € A, there
isbe B satisfying zy = byz.
Prove that A is commutative.
Solution. As k and p are mutually prime, by Euclid theorem there are
a,b € Z such that ka+pb =1, that is (k1 4)(ala) = 14. For 7 =y = 14 we get
14=beB.
Suppose A is non-commutative, that is, there are 7,y € A, b € B\ {14} 2.2. THE NATIONAL MATHEMATICAL OLYMPIAD
such that zy = byz. Thus z + k14 and y do not commute for k
As B\ {14} consists of p — 1 elements, there are s < r in {1,2,...,p— 1} and Final Round - Solutions
@& B {T3}, such that (z + r1a)y = ay(z + r14) and (z +s14)y = ay(e + s14).
Denoting by z = z+ s1a and t = r —s & {1,2,...,p~ 1} we get zy = ayz and
(2 +t1a)y = ay(z + tl15). This gives y = ay, or zy = ayz = y=. In conclusion
(2 + 51a)y = (z + s11) implying =y = yz, a contradiction.
7" GRADE
PrOBLEM 4. Let a,b € (0,1) and let / : [0,1] — R be a continuous function,
such that PROBLEM 1. On the sides AB and AD of the rhombus ABCD, consider the
. - - points E and F respectively, such that AE = DF. Lines BC and DE intersect at
P, and lines CD and BF intersect at Q. Prove that:
/0 f(L)sz/u !(t)dH»/a F0)d, for any 7€ 0,1] PE _QF
O IE+ o
a) Prove that a + b < 1 implies f b) Points P, 4,Q are collinear.
b) Prove that a + b = 1 implies that / is constant.
Solution. a) As / is continuous, the functions in z defined by the given i a) From BE | CD2D wewe have &PE
(;:)m;?;. -5 = _BE
22
&z, and and from from DF ||| BC we
equality, have finite derivatives. Taking derivatives on both sides, we get &t G5 = por These mply
f(z) = af(az) +bf (ba),
PE _QF
LA 1
for all 2 € [0,1]. Consider M = sup,cgoy [£(t)]. By the mean theorem, we have P07 QB
1£(@)| < (a+b)M, that is M < (a +b)M. Thus M = 0 implying f
b) Let 2o be a maximum point of /. Then
b) From FD | BC, we deduce 0B~ B~ B Thales theorem gives
f(zo) = af(azo) + bf(bea) < (a + b) f(o). EF | AQ
It follows that azy is a maximum point too. By iteration, a”zp is a maximum point, From BE | CD, we get ? ICLE: %. and by Thales theorem we
for any n. As f is continuous, and_lim a"zo = 0, we infer that 0 is a maximum deduce EF | AP. Tt follows that the points P, 4,Q are collinear.
point. In thesame way, if ) is a minimum point for 7, we get by induction that
4"z, s & minimum point, for any n. Considering limits, we conclude that 0 is & PROBLEM 2. The side-lengths of a triangle are a,b,c.
winimum point. As 0 is in the same time maximum and minimum point for f, we a) Prove that there is a triangle with sides /a, v/b and /2.
get f(z) = £(0), for all2 € [0,1], thus £ is constant. D) Prove that vab + Ve + Vac < a+ b + ¢ < 2vab + 2vbe + 2/ac.
Solution. a) From a | b > ¢ one obtains a b + 2v/ab > ¢, or (Va+vb)? > e,
that is \/a@+ vb > \/&. By symmetry, this proves that \/a, b, / are the sides of
a friangle
b) The inequality vab+v/be+/ac < a+b+cis equivalent to 2v/ab+2vbe+
2y/@c < 2a+2b+2¢, or (a —2vab+b)+ (a - 2y/ac+c) + (b —2vhe+¢) > 0, that
is (va—vB)? + (va— v2)? + (Vb — v/e)* 2 0, obviously true.
52 Souurions Sotumions - FiaL Rousn 53
) Prove that one may choose 6 points such that no isosceles triangle can be
Because /a, v/b, /2 are the sides of a triangle, we have: drawn with vertices at these points.
b) Prove that one cannot choose 7 points with the above property.
Va<vb+ve Vh<ya+ve ve<va+vh
Solution. a) Any of the following configurations has the asked property:
Multiply the above relations by /a, v/, v/, respectively. Summing up we get
a+b+e < 2v/ab+2vhe + 2y/ac.
PROBLEM 3. The diagonals of the trapezoid ABCD are perpendicular and
intersect in O. Angle A equals 90°, AB | CD and, AB > CD. The diagonals
intersect at O. OF is the bisector line of the angle AOD, E'is on the segment AD, b) Suppose that we select 7 points such that there are no sosceles triangles
and F is on BC, such that EF | AB. Denote by P, Q, the intersection points of bavin the vetics at these poins. There are exactly o possbiltes:
the segment EF with the diagonals AC and BD, respectively. Prove that: All chosen points are on the border of the big square. Partition the set of
a) EP = QF; 12 pmms in three sets that are the vertices of three squares as seen in the figure:
b) EF = AD.
Solution. a) The triangles EPA and DCA are similar. We get

codo
EP _AE +
® G~ 4D
Observe that we cannot choose three points with the same symbol that are not
From the similarity of the triangles FQB and CDB we also obtain
the vertices of an isosceles triangle.
QF _BF 2) There is at least an interior chosen point. With the other 15 points form
@ DG~ BO five sets as seen in the figure:

@oex

®o®o
From the points marked with o, ®,, we can choose only one point and from those
marked with », we can choose two points. In total we can choose at most 6 points.

8" GRADE
AE75 = 5_BF and from (1) and (2) e obtain 7EP = 7z
By Thales theorem _QF
Prosiem 1. Find all non-negative integersn such that there are integers a
that is EP = QF. and b with the property:
b) Because ZA0Q = LAEQ= 90°, the quadrilateral AEOQ s cyclic, thus
LEQA = LEOA) = 45°. Tt follows that the triangle EAQ is isosceles and AE n*=atb and n®=a’4 b7
The quadrilateral DEPO is cyclic because m(DEP) = m(DOP) = 90° Solution. From the inequality 2(a? +¥7) > (a+b)? we get 2n° > nf, that
This implies £ DPE = /DOE = 45°, that is DEP is isosceles and DE = EP. isn<2 Th
But EP = QF. We get EF = EP + PF = ED + EA = AD.

PROBLEM 4. Sixteen points are placed in the centers of a 4 x 4 chess table ~ for n = 2, we may take o
in the following way:
PROBLEM 2. Prove that the equation
2t byttt = 220
54 Sovurions SoLutioxs - FinaL Rouxn 55
where 0 < = <y < = <#, has exactly two solutions in the set of integers. The triangles B,C1C and ABC are isosceles and B,Cy = OCy, AB = ACy. Thus
BiCy + BC = AC.
Let M € CB (B € CM) be such that BM = B,Cy = B'C’ and N is the
‘midpoint of AM. We find out that MB'C’B is a parallelogram, thus MB' = BC,
and MB' = AM.
In the triangle ACM we got CN L AM, and DyA L AM from (1). We see
23y, ¢ = 20y, where 0 < z; < that AM is perpendicular on the planes (CB'B,) and (AD'Dy), thus the segment
502, By the same argument AN equals the common perpendicular of the diagonals CB and AD'. It follows
223, 1= 2, Where 0 < 22 < 4 < 2 < tp are integers AN = 1AM = {BC".
Recursively, = Y = 220, 2 2001, ¢ = 32014, where 0 < a < PROBLEM 4. A cube of side 6 units contains 1001 unit cubes with sides
b< e < dare integers and a? + b2 + c? 4 & 4. This relation simply implies the parallel to those of the given one. Prove that one can find two unit cubes such
conclusion. that the center of one of them is inside or on the faces of the other.
PropLEM 3. Consider a frustum of a regular quadrilateral pyramid Solution. Partition the cube into 12* cubes of sides § each. The centers
ABCDA’B'C'D' in which the lines BC' are DA’ orthogonal. of the 1001 unit cubes are at least } apart from the initial cube faces. As a
a) Prove that the angle between the lines AB' and DA’ equals 60°. consequence, they cannot be inside the cubes of side § which have a face on one
b) If the projection of B’ onto the plane (ABC) is the incenter of the triangle of the faces of the given cube. It results that the 1001 centers are inside the 1000
ABC, prove that the distance between the lines B’ and AD" equals 1BC". cubes of side } which are inside the given cube. By the pigeon-hole principle, two
Solution. a) The diagonals BC' and DA’ are contained in perpendicular of centers of the unit cubes are inside or on the faces of the same cube of side
planes to (ABC). They are parallel and contain the points B and D, respectively. This proves the result.
The diagonals CD' and AB’ belong to planes which are perpendicular to (ABC)
and contain C and A respectively.
9" GRADE
PROBLEM1. Find all increasing functions £ : {1,2,...., 10} — {1,2,...., 100},
Dy c having
the property: 2-+y is a divisor of (z)+yf(y), for any z,y € {1,2...., 10}.
D ¢, Solution. As z +y is a divisor of both zf(z) + yf(y) and =f(y) + yf(y),
we get by subtraction z + yla(f(y) + F(x))
For y = 2+ 1 e obtain that 22+ 1 is a divisor of &((z + 1) — f(z)) and,
NG, because (22 +1,) = 1, we deduce
In particular f(z+1) ~ f(r)
2z + 1|f(z +1) ~ /(z).
> 20+1, for all 2 € 1,2,....,9, thus
A B s o
Iy S+ 1) - 1@) > 3 +1).
M = =
Consider a point in the space and draw through it parallel lines to the diag- ‘This implies £(10) > £(1) +99 > 100, thus £(10) = 100 and (1) = 1.
onals BC', C', DA’ and AB' respectively. One obtains a regular quadrilateral From f(z+1)—f(z) = 2z+1, forany ¢ = 1,2,....,9, we conclude f(z)
pyramid; its diagonal section s an isosceles right triangle (thus equal to “half” of
the base square). PROBLEM 2. For any positive integer n denote by P(n) the number of
We infer tiat the lateral edges of this pyramid are equal to the base edges, quadratic functions /: R — R, f(x) = ux” + bz + ¢, haviug tle properties:
‘meaning that the lateral faces are equilateral triangles. We thus get that the angle 2) ayb,ce (L,2,...nk
made by DA’ and AB’ cquals 60°. b) The equation (z) = 0 has integer roots.
b) Let BiC) be the projection of the segment B'C” onto the plane (ABC) Prove that n < P(n) < n?, for all .3 4.
and Dy the projection of point D’ onto the plane (ABC). Then Solution. As the quadratic equations = + ke +k—1 =0, k = 2,3,...,n
(1) CBy | AD; and 227 +4z+2 = 0, 27 + 4z + 4 = 0 have integer roots, we infer P(n) > n,
56 SoLutions SoLutions - Fival Rousp 57
For f satisfying the given conditions, we write f(z) (z+a1)(z +aa) with Let Mo be the tangent point, and wy its center. The lines woMo and HC'
1,09 €2, 0, alay + ), acyan € {1,2,...,n} are parallel (they are both perpendicular on BC'). As a consequence LuoMoH =
In pasticular, oy < ——.
o1 We find LMyHC'. But LwoMoH = LwoH Mo, implying that HMo is the interior bisector
ine of /BHC"
Tet S be the point corresponding to this configuration. The locus will be
the segmet AoSo situated on the line HAq, and the limiting position Ao will be
obtained when d coincides with one of the perpendicular lines BB’ or CC".
Remark: The preceeding discussion implies that when d rotates around
Asi+b+d+ H, remaining inside the angles /BHC' and /CHBE', the point S moves along
conclude P(n) < n’. the segment AqSp in two ways: to both lines dy = MyHNy and dy = MaHNy,
My, M; € BC' and Ny,Ny € CB', that make the same angles with the bisector
Remark. A more careful analysis of the above solution implies a better line HMy of ZBHC", there carrespond the same points P and @, that is the same
bound, namely P(n) < flT forn zd. point 5.
PrOBLEM 3. Let H be the orthocenter of the acute triangle ABC, and let PROBLEM 4. Let p,q be positive integers, p > 2, ¢ > 2. A set X has
B',C" b the projection of the points B and C' onto AC and AB, respectively. A property () if, by definition, for any p subsets B; C X, i = 1,2,...,p, not
variable line d, passing through H, intersects the segments BC' and CB' in M necessarily different, any of them having g elements, there is set ¥ C X having
and N, respectively. The perpendiculars in M and N on d, intersect BB and P elements, such that the intersection of ¥ with each By, i = 1,2,...,p, has at
CC',in P and Q, respectively. most one element. Prove tha:
Find the locus of the midpoint of the segment PQ. a) Auy set X with pg — g elements does not satisfy property (S);
b) Any set X with pg ¢ + 1 has property (S)
Solution. By the following similarities: Solution. a) If X has pq — g = (p — 1)g clements, chose p — 1 subsets B,,
AHMP~AHB'N, AHNQ~AHC'M, AHBC'~AHCE', 1p — 1 with g clements each. They form a partition of X, and let B,
arbitrary, containing ¢ elements. By the pigeon-hole principle, any subset ¥ with
P elements, will intersect at least one of the sets B, i = 1,....,p— 1 in at least two
elements,
1) o given , bserve that ) By contas ot most (9 = 1)g lements. 4
X has pg—q 41 = (p~ 1)g + 1 clements, ve will ind at least one element at our
disposal, say z;, which does not belong to any B, j # i
Apply the above remark for i = L: if 21 € B; we will continue; if not, we
replace an element, say 1 € By, by 1. Continuing with i = 2, 3, etc., we observe
that at each step the chosen z; will be different from all the preceding ones.
Consider Y = {zi | i = 1,....,p}. The set ¥ will intersect each new B in
exactly one element, that is 7;. Replace now z; with y;, to obtain the original B;
we obtain 21F. ; HM HQ HC HP _HQ
Therefore 77 = 77i % Ty ¢ Y, the intersection of ¥ with B is empty; if y, € Y, the intersection would
7 contain an element, that has to be y.
As ncgv(‘;cquenrfl; themen B w5 a el Thus, 2 middSiac
S of the segment PQ is situated on the fixed line H Ao, where Ao is the midpoint
of BC. 10" GRADE
Conversely. let S be a point of the Jocus and PQ the corresponding segment.
The corresponding point M is one of the points My, My, where the circle of di- PROBLEM 1. Let f : R — R be a function, such that |f(z) - (y)] < |z — yl,
aneter 1 ntrsects the segment 5. The ine Mulf will interect Gt Ny for any real z,y. Prove that if the sequence , f(x), f(f(x)),... arithmetic, for
k=1, The perpendicula ine . on MiNo will ot OC" . Qu, b any real z, then there is a real a such that f(z) = z + a, for any real number z.
frst pmt of the proof, PQs | BC, k = 1,2. But PQ | BC, so Qi
Therefore, we find out that one of e limiting configurations corresponds to the Solution. Suppose the existence of 7,y & R such that f(z) ~z > f(3) -
case when the circle of diameter HP is tangent to BC" Denoting by p = f(z) —z, and g = f(y) - y, we get f(z +np) = = + (n + L)p,
58 Sorurions SoLuTions - FinaL Kousd 59
fly+na) = y+(n+1)g, for all n € N. As we must have |f(z +np) — f(y +ng)| < which implies
Ja+np—y
— g, we deduce [z —y + (n + 1)(p—g)| < |z ~y +n(p—q)] -
For suficiently large n, we will have z ~'y + (n + 1)(p—g) < £~y +1(p )
implying p < g, a contradiction. Thus f(z) —z = f(y) ~ y, for all z,y € R, that > logy(n? ~ logy(2logy ) = 2logy n+ logy (1 -log(2logy n)
is f(z) ~ = = £(0) ~ 0, a constant. 2
PROBLEM 2. Prove that any thetrahedron in which pai of opposite sides We have to prove that
are equal and make equal angles, is regular.
Solution. Denote by ABCD the given thetrahedron. We shall use the logy(2logy n)
following obvious vectorial relation logy (1 -eallan), 21
AB.CD+ AC- DB+ AD - BC =, or
Let a the common angle between opposite edges. We get log@2log " 1
(l C: >3
cosa- (£AB-CD+ AC-BD + AD-BC)
=0,
2logyn) >3
Using the Bernoulli inequality, it is sufficient to prove 1 — %n“—)
_ 1
for some choie of thesequence o + - signes
Tt cosa7 0, Then £AB -CD 4 AC" BD & AD - BC = 0, and ve find that that is log,(2log,
@ n)
<3 1
one of the numbers AB-CD, AC - BD, AD- BC equals the sum of the other two,
contradicting the Ptolemei relatior The last inequality is equivalent to 2logyn < 2%, or logyn < 2°F. It is a
Thus,cosa 0, that i oppesie s sceoxthogonal. It s an casy argumont
to show that AB? + CD? + BD? = AD® + BC? (take for example AD' routine induction argument to finish the proof.
perallel and equal to BD . observe that triangles ACD' and DCD' are right
triangles). As by the hypothesis AB oD, AC BD and AD = BC, we PROBLEM 4. Let (P)az1 an infinite family of planes and (Xo)>1 a family
conclude that all edges are equal, so ABCD is equilateral. of non-void sets of points such that X, C P and the orthogonal projection of
Alternative solution. One can easily prove that 2|cosa| - AB - CD = Xot1 onto the plane P, is contained in X, for any n.
|AD? + BC? ~ AC? — BD?|. Denote by z,y, = the edges, we get Prove that there is a sequence of points (py )3y such that p, € X, and p,
is the orthogonal projection of a1 onto the plane Py, for any n.
Does the result remain valid if the sets X, are infinite?
which implies Solution. Consider positive integers n,p. Denote by An,p the image in P,
of the set Xius, after succesive projections onto the planes Prgp-1, Pa: Any
PROBLEN 3. Let n > 2 be an integer and a & (0, 00) such that s a non-empty subset of X, AS Xsp41 projects onto the plane Py, as a subset
Of Xugp, we deduce that Anpi1 C Ay For any n we get a decreasing sequence
2 +logya =n? of non-empty subsetsX D Ay 2 Anz D+ D Any 2
Prove that As X,y is finite, one can find p > 1 with the property that A, = Ay, for any
2|og27l>u>2]m1n71 k> p. Denote by T, this subset of X.. For large p, Anp =T, Ans1p-1 = Tuins
thus T, is the projection of Ti;1 onto the plane P,. Chosa an arbitrary ps in 7.
Solution. Tt is clear that a > 1, 50 7% = 2% + logya > 2%, which gives Then, there is pz in T, such that p is the projection of 2 onto Py. Inductively,
2logyn > a. construct the sequence (pa),5y stch that pn € T, pus1 € Ty and such that py
In order to prove the second inequality observe that is the projection of pas1 on' Py, This concludes the proof.
1F the sets are infinite, consider the planes P, with equation z = n and the
n? = 2% +logya < 2° + log(2logym), sets X = {(,0,m) | = 3 n}. It is easy to check that the conclusion fails
60 Souurions Sourmions - FiaL RouD 61

11" GRADE When n = 2p + 1, choose Azpsy = Agr 8)e o= (% §)


PROBLEM 1. Consider an integer n > 3 and the parabola of equation y? b) Suppose rang (XY) > rang (YX), We have
2pz, with focus F. A regular n-gone A Az - Ay has center at F and no one of its
vertices lies the 7 axis. The rays FAy, FAs, .., FAy cut the parabola at points 6} rang (XY) — rang (Y X) < rang (XY) < rang (X).
By, By,..., B,
Prove that FBy + FBy+ -+ + FB, > np. From Sylvester inequality, rang
(Y X) > rang (X) + rang (V) — n, which im-
Solution. Let FBy = t, a the angle made by FB; with Oz and let a plics rang (XY) — rang (¥ X) < rang (V) + n — rang (X) — rang (¥), that is
a+ 25207 1t follows that the coordinates of B are (§ + i cos o, ti sin av). 2 rang (XY) — rang
(¥ X) < n — rang (X).
We have # sin? ax. = p? + 2ptx cos ax. The positive root of the last equation
We get: From (1) and (2) we get rang (XY) — rang (Y X) [ ]
n
ProBLEM 3. Let f : (a,b) — R be a function with the property that for any
2 € (a,b), there is a non-trivial interval [az, b:), a < a; < = < by < b, such that f
is constant on (¢,b,]
Using the Cauchy inequality we obtain ) Prove that the image of f is a set which is finite or countable.
n b) Find all continuous functions satisfying the given property.
(ED(Ew)
= ) >n?,> Solution. a) Let f(z) € Im f, with = & (a,b). Choose . € @1 [ax, bs]. By
the given condition, f(x) = f(rz). Thus Im f = {f(rs) | = € (a,b)}.
1€ we denote {2 | @ € (a,6)} by {102, 1Gn.-.}, then the set Imf =
that gives 3t > np. {7(gn) e 5 at most countable.
The equality implies ¢ t, =7, which is impossible because the ') 1f £ is continuous, we shall prove that it is constant. Suppose that f takes
circle of enter F and radius r intessets the parabola in at most two poins. at least two different values, say A and p, with A < . The function f has the
intermediate value property, that is [\,4] C Im f. This is in contradiction with
Remark. Tt can be shown that 3t = ——"-2__ the fact that Im is countable.
PROBLEM 4. ) Construct a function f : R — Ry with the following prop-
PROBLEM 2. Consider an integer n, n > 2, erty, called P: Any z € Q s a local strictly minimum point for f.
) Prove that there are matrices A, B € M,.( ') such that b) Construct / : @ — R, with the property that any point is a local strictly
‘minimum point and is unbounded on any set of the form 1 Q, where I is a
rank (AB) — rank (BA) = [g] non-trivial interval.
©) Let f : R — Ry be function which is not bounded on any set of the form
b) Prove that for any X,Y € My (C) we have 1N @, where I is a non-degenerate interval. Prove that f has not the property P.
Solution. a) Define
rank (XY) — rank (VX) < ['5‘] .

Solution. ) Let A = (0 n) and (g é) One can easily check that


sang (AB) — rang (BA)
In the case when n 2p, choose One can see that z = is a local strictly minimum point, because there is
a 0 B o an open interval around z that doesn't contain fractions with denominators in the
set 1,2,....p.
b) The function / : @* — R defined by f (g) = p has the given propertics.
62 Sovumions Sowumions - FixaL Rousn 63
) Suppose that f has property P. Consider a non-degenerate interval Iy As lim ¢, =z, the continuity of /7, and (3), give together
[ao, Bo]. By induction, we shall define a decreasing sequence of closed intervals
I = [ow. B, k= 0,1, such that f(Ii) C [k,o0). Suppose we have constructed
I,Lo, . , Ty with the given properties. We can find 7, € QN [_1 with f(z,) > £1(2) = lim 1"(62) =0.
n. Take a of Zn, In = [an,Bu] C -1, such that for z € I, f(z) > f(zn) > n.
This proves the induction step. As @ was arbitrary, we deduce that f has the form f(z) iz +b, where a,b € R.
From the fact that ax s increasing, i is decreasing and ay, < B, we deduce Substituting in the given relation, we find a = 1.
the existence of an element 7 € () I, that is f(+) > n for all integers n, a We conclude f(z) = z + b, with b arbitrary.
contradicti PROBLEM 2. Let f € Z[X]. Forn € N, n > 2, define f, : %y — Zy, by
I (), for any z € Z.
12 GRADE Find all polynomials f € Z[X] such that for any n € N, n > 2, the function
Ju is onto.
ProBLEm 1. Find all continuous functions f : R — R satisfying Solution. a) If in Zn, we get k € Z such that
= —y = kn. We can
write
e A fl2) = fy) = (= = w)g(=,9),
” A Sdt=ni@)+5,
where g(z,y) is an intoger. Thus f(z) — f(y) is & multiple of n, and as a conse-
for any z € R and any positive integer n, n > 2. quence, () = F(u), Le. fa(3) = fu(d)
b) First, ve show that deg /'S 1. Suppose, by way of contradiction, that
Solution. As f is continuous on K, consider an anti-derivative F. The given deg f = k > 2. The polynomial 9(X) = f(X + 1) — £(X) has degree k— 1. As
relation is equivalent to k=11, one can find = € Z, such that |g(z)| > 2. Consider n = |g(z)| =
1fe+1) = @) o
" i (p(e 2) -re0) @)+, In the ring Zy we have f(z +1) = f(z), or fu(¥+1) = fa(%). The last
equality shows that f, is not injective and as a consequence nor onto. As f can
not be constant, let 7 € Z*, o € Z, such that for any & € Z, f(z) = mz + a.
for any 7 € R and all n € N* We shall now show that [m| = 1; for if [m] > 2, the map fim| : Zjm) — Zjm
As a consequence, f has finite derivative at any point. Taking derivatives in s constant, thus not onto, a contradic n.
(1), we get It s easy to see that the polynomials X + a and —X + a satisfy the given
conditions,

@ »(/ ('t+%>7/(x =1 PROBLEM 3. Let : [0,1) - R be an integrable function such that


forz€Rand n e N*. N .
From (2) it follows that f is twice derivable and the following relation holds / f@)dz ,L 2f(@)dz=1.
Prove that
n (f' ( + %) -1@) =@,
L’ Fladz > 4.
implying that f" is continuous.
Consider z € R. By Lagrange theorem, relation (2) implies the existence of Solution. The degree 1 polynomial p satisfying the relations [} f(z)dz =
a point ¢, = cn() € (¢,7+ 1) with f(c,) = f'(z). By Rolle theorem, for /' on
the interval with endpoints ' and c,, we get a point belonging to the inferval Ja #f(z)dz = 1,is p(z) = 6z — 2. Thus
with endpoints z and ¢;, and having the property that
®) S"(G) [to - npas = [o atrt@) -y az =0
64 Sownons
We get

This implies
1 1
USL (f(x) ~~ p(x))" p(a) ?da o = /0 F(@)(f(z)(x) =- p(a))® p(z) ? & dz 2.3. THE NATIONAL MATHEMATICAL OLYMPIAD
[) fAz)de -6 l;(l)mu/‘ fla)dz = /;‘jz(x)dz s Selection Tests for the BMO and IMO 2004
whence the conclusion.
PROBLEM 4. Let K be a field of characteristics p, with p = 1 (mod4). First Selection Test
Prove that any non-zero element in K can be written as the sum of three
squares of non-zero elements from K. PROBLEM 1. Let a1,a2,a3,a4 be the lengths of the sides of a quadrilateral
Solution. Let m € N* such that p = dm + 1. and s be its semi-perimeter. Prove that
a) By Wilson theorem pl(p ~ 1)! + 1, thus (p— 1)! +1 = 0 in K. This
gives (4m)! = —1in K. On the other hand 1 = ~dm,2 = —(4m — 1),...,2m = 4 12
Z(@m+1) in K, giving Xa<s
sta i
~1)2™
(dm)? (4m — 1) - (2m + 1)%
‘When does equality hold?
We get.
[(dm)(dm — 1)+ @m + 1)), Solution. Numbers s — a; are positive, and the AM-GM inequality gives
in K. We conclude that 1 is a perfect square in K.
b) Consider a,b € K* two commuting elements. We have o 15> (Te-a) "
i i
&) (a+b)?
= a? 4?4 2ab
for everyi = 1,2,3,4. Since X(s —a; +a, relation (1) leads to
and i
@ (a—b)?
= o 45 — 2ab, @ o< (’[#1‘4)’”.
Consider b=2"1. If a € K then a and 2~ commute.
Ta# -2, from (1) we get a = (a+271)2 - a2 42 (a+2” )24af 8, On the other hand, the AM-GM inequality gives
because —1 is a perfect square in K.
Tra=—2-1, thena # b. By (2) we havea = a?+b — (a—b)? = a? 467
4 ¢%, 1oy 1 1 )l/z
because —1 is a perfect square in K.
Thus any a € K* can be written as a sum of three non-zero squares of some [P cern
e i (K}L, Toa=a
elements in K. @) L e
(M=)
# g
Combining (2) and (3) and adding we get the required inequality. The equality
takes place if and only if the quadrilateral is a rhombus.
66 Sovutions SowvTions - SkLEcTION TesTs 67

Alternative solution. We can write, using AM-GM-HM inequalities get £(0) = 0 and if we suppose f(k) =k for any k <, from f(n+1) <n+1and
the injectivity condition, we get f(n + 1) =n+ 1.
1 4 1 Suppose, by way of contradiction, that f() > n for some non-negative n
Goal-a) 2620‘”, = () and denote f = [0 f oo f, the k-composition of f with itself, k = 2,3,....
We easily obtain by induction that f¥(n) < f(n) for any k = 2,3,....
Fr(n) <nforp=1,2,....k we have
Indeed if
29 L1
25 mn Dra o < SO0 g,
PROBLEM 2. Let R, i = 1,2,....,n, be pairwise disjoint closed rectangular Thus the sequence (f*(n))y is a bounded sequence of non-negative integers and
regions whose sides are parallel to the coordinate axes. It is also known that the S0 f#(n) = f9(n) for some p < q. Since f Is injective, it follows that f2-#(n) =
area of R = U Ry is at least 4 and the projection onto Oz of their union is an therelore f3-7+{(n) = f(n), a contradiction.
It is obvious that the identity function verifies the given relation and is
nterval. injective.
Prove that R contains three points which are the vertices of a triangle of
area 1. Remark. There are many other ways for proving the result and maybe the
Solution. Let [a,] the projection of R onto Oz. For z € [o,b], define simplest is to make connections with a well-known fact on imits of sequences: if &
sequence of non-negative numbers, say (an). is sub-additive, that is
m(z) = min{y| (z,y) € R} and M(z) = max{y| (z,y) € R}. Since R is included
in {(z,y) | € [a,b], m(z) < y < M(z)} and has area at least 4, it follows that s € 2L for any n, then the sequence has a finite limit.
there exists #p € [a, ] such that In our case, for any n the sequence (f(n)), is sub-additive, and being
a convergent sequence of integers is constant beginning from some term. This
(b~ a) (M(zo) —m(z0)) > 4 contradicts the injectivity, as above, if f is not the identity function.
Consider the points A and B having coordinates (zo,m(z0)) and (zo, M(z0)) PROBLEM 4. Consider an integer n > 2 and a dise D in the complex plane.
respectively, and suppose, for instance, o < "T“’ Then, for Prove that for any 1,22,z € D, there exists z € D such that 2" = 2125+ 2.
Solution. First, consider the case n = 2. In this case, if u? = 2125, then the
RS
Ve e two complex roots of second order of 21, 23 are u and satisfy
o=l 2z +ul+ o1 +ul 22—l 2l a1 = zal,
we have 7y < 71 € 7 + %5% < b, therefore one can find a point C € R having
coordinates (z1,y1) such that area(ABC) which shows that 21,, 22, ~u are (in this order) the vertices ofa cyclic quadrilat-
eral. This shows that u and —u are on the two arcs determined by z; and z; on
Remark. The above solution is an elementary version of the obvious one acircle C; since at least one of these ares is included in D, it follows that u or —u
given in terms of the Fubini theorem for a double integral. It is thus clear that idis D,
the fact that the rogions are rectangles is not important, the main point being the In the case n > 3 we may suppose that s = [a¢ # 0 for k = 1,2,...,m,
interval-projection property. and we will denote 7 = /72T An argument based on continuity (there
s the author pointed out to us, the problem comes from a conjecture of exists w, on the segment [z, 2] such that fusl? = 1] |2, there exists wz on the
Erdlos. segment, [uy,5] such that [ua]® = |uwy[?]zs], and so on shows that there exists at
least; one complex number of modulus 7 inside D, therefore the intersection of the
ProBLEM 3. Find all injective functions £ : N — N such that, for each n, circle C(O, ) and the interior of D is not empty.
1£.C(O.7) is included in D, then every root of order n of 2,2 = belongs
i) € —5
(n) toD.
1£ C(O, ) is not included in D then, for some sufficiently small interval (a,5)
which contains r, the intersections of D with the circles C(0,a) and C(O,b) are
Solution. It is clear that if we prove that f(n) < n for any non-negative n ares Ay,Ap of these circles. Let A be the region of D situated between Ay and
we shall get by easy induction that £(n) = n for any . Indeed, from f(0) < 0 we g, that'is A= {z € D a < |2| <b}. Then there exist , f € R such that every
SoLutioxs - SeEcTioN Tests 69
68 SoLutions
Then, all the sectors corresponding to ax are of the same color (otherwise axs1 is
2 € A has an unique argument between o and f. The case n = 2 and the lemma less then ax), for instance red.
below allow replacing in a finite number of steps (1,22, . , zn) with w; € A for Itis now easy to see that the half-disc starting “just after r¢” and taken clock-
G=12,...,nand 21252 = wywy - wy. Then we can t wise fulfls the requirement. Indeed, if this half-disc contains p red sectors and n—p
blue sectors, then, with indices taken modulo n, it COMtains T441,Tks2, s Thip
and by, bt onipt1, hence it contains all numbers from 1 t0 1.
z=r{cosy +ising), ¢
Propuen 6. Find all non-negative integers which can be reached by the
expression
Lemma. A transformation ofa family (3,72, ..,a) of positive numbers @ tabii?
consists of replacing r; = max{ry|p = 1,2,. .,n} andrj = min{rylp=1,2,....,n} @-1
with /777 and /7575 when a,b are non-negative integers and ab # 1
Ifa < r= §/rra7 < b, then, after a finite number of such transforma-
tions, we get a family (s1, 52, -, 8n) With 8, € (a,) for p=1,2,...,n. s
Proof. Let a =Ina,§ = b,z = lor and 2, = Inr —p, p = 1,2,...,n.
Solution. Let SEWHE@1 ey
Denote d, the value of the sum ¥ [z, — 2 after s transformations. In the case a = b, the number & = 3+ —— is a positve integer if a =
We suppose, for sake of simplicity, that 7, < -+ < 2k < z < Tkt < or =2, whence k = 0 or 4
Zp. Then (& — 1) + -+ + (& — 2) = (Tkpt — 2) -+ + (zn — 7) and
n

The case b= 0 leads to k = —a?, hence a = 0 and & =0.


We will look now for the solutions (a, ) such that a > b > 0. Since the given
relation is equivalent to a? — (k—1)ab+b?+ k = 0, we sce that if (a, ) is a solution
and b > (k — 1)b—a >0, then (b, (k — 1)b— a) i5 also a solution. The inequality
(I~ 1)b—a > 0 is true in all cases because it becomes successively
), dy is modified by
atb
k>2, altabt¥
TEETT atd
L0 B> —a-b.
b a1
= ~2min{z — 21, 7, — 7}
Tn the same way, the inequality b > (k — 1)b — a is successively equivalent to
But d, < 2n(z ~ 22) and d, < 2n(za ~ 2), implies 2min{z
— 21,70 ~ 2} > ., k B 2ta
Te a’+ab+¥
etT _+a
3o a>b
£ for > b>1
o
whence dyy < 2= 2d,. This shows that (d,), tends to 0, and the conclusion
follos immediately. 1£ 35 < b2 - 1, that is b > 4, the previous inequality is true, therefore for each
solution (a,b) with a > b > 4 we find a solution (b,c) with b > ¢ > 0. Thus, the
descent method shows that each solution corresponds to a solution with b < 3.
Second Selection Test Forb=lwegetk=a+2——7 a=dora=2,
PROBLEM 5. A disc D s divided into 2n cqual sectors, n of them are colored For b=2wegetdk=2a+5+
E 5
+ +7“§|‘21 a a=1lk=dork
ora=1
in red and the other n are colored in blue. Starting from an arbitrarily chosen
sector we count; from 1 to n, in a clockwise order, the red sectors. We proceed in For b= 3 we get Ok = 3a+ 10 + —— which does not yield any solution.
the same way with the blue Sectors, but in an counterclockwise order.
Prove that there exists a half-disc of D which contains all the numbers frou Thus, the auswer is k € {0,4,7}
Tton
PROBLEM 7. Let a,b,c be integers, b odd and consider the sequence zo =
Solution. Let r, and respectively by the red, respectively the blue, sector 4,21 =0, 77 = 2,2
which carries the number i =1,2,..., 1.
Consider for each i the are a;, taken in a clockwise order, which starts “just Tp = g +bra_a + erug, for n >4,
after r; *, and ends “just before b;" and assume that the smallést of the a;-s s ai.
70 Sowurions Sovurions - SeLEcTION Tasts 7
Prove that if p s a prime and m a positive integer, then 2, is divisible by p. is also divisible by p, because S is also a symmetric integer polynomial (hence
Solution. Consider the characteristic polynomial S(a1,03,a0,01) is an integer)
f=X*=eX*~bX-a PROBLEM 8. A square ABCD is taken inside a circle 7. Inside the angle
opposite to £ BAD is taken the circle tangent to the extended segments AB and
associated with the given recursion formula. We can check that all the roots of f AD and internally tangent to 7 at Ay. The points By, Cy, Dy are defined in the
if ais a 100t for both f and f' = 4X—2¢X ~b, then a is a oot of g same way.
—2¢X?~3bX ~da, foot of h = ¢’ +2Xg = —6bX?— (2¢>+ 8a) X —be Prove that the straight lines AAs, BBy, CCy, DDy are coneurrent.
g — ch = (2¢* + Sac — 98%)X + be? — 12ab. Since I(a) = 0 and Solution. The common point of Ay, BBy, CCy, DD is the center P of the
26 + 8ac — 95” is odd, it follows that o s rational and, because / is monic and homotethy H of negative ratio which sends the incircle of ABCD into 7.
F(@) = 0, we deduce that o is an integer, whence f'(a) = 4a® — 2a — b is odd, a
contradiction.
The above result shows that there exist cocfficients a1,az,as,as, uniquely
determined by 2,1, 72,3, such that
2, = a0f +aa} +asa +asaf,
where s, 03, a3, & are the roots of f. We notice that
zo=4=al+af+a+al This follows from the fact that the circle 71 tangent to v at Ay can be
1 a1 +az +as +aq obtained from w using an homotethy H; of center A and negative ratio, and 5 can
af+aitafial=-2Tma be obtained from ; through an homotethy Hj of center A; and positive ratio.
of + o +of +of =8 anenas, Therefore H = H; 0 H} has its center P on AAy. In the same way P belongs to
which proves that a; = ag = BBy, CCy, DDy
Remark now that if p is a prime, then p| (’;) for each k = 1,2,
hence Third Selection Test
(X+YP=XP 4 Y7 4pQ(X,Y),
where @ is a symmetric integer polynomial in two variables. This easily extends PROBLEM 9. Let n > 1 be a positive integer and X be a set containing n.
to elements. Ay, Ay, . , Ao, are subsets of X such that the union of any 50 of them
(X4Y 4+ Z4TP = XP +Y? + 27+ T7 + pR(X,Y,
Z,T), has more than $n clements.
where R is a symmetric polynomial in four variables. Prove that among the given subsets it is possible to choose three, such that,
In order to prove the required conclusion, we shall use induction on m. For every two of them have a non-empty intersection.
. we notice that Solution. Consider a graph T with vertices Ay, A, Asor and edges be-
tween the vertices whose intersection is non-empty.
5=l
+08 +04 + o = ~pRlan, 3,03, a), 1f this graph has no triangles, the it has 51 vertices with degree at most 50.
Tndeed, if there are at most 50 vertices with degree at most 50, then there are 51
and since R can be represented as an integer polynomial in symmetric fundamental vertices with degree at least 51, o two of them, say A and B, must have a common
polynomials, R(a, s, s, a) is an integer. edge. Since A and B are connected each with 50 vertices among the remaining
Assume now that z;m i divisible by p. Then 99, there exists a vertex C connected with both of them. We thercfore obtain a
triangle ABC, which is a contradiction.
g = g™ v 0™ v 0™ Denote now by Ai,, Ai, ., As, the 51 vertices of degree at most 50. Each
of the A;, has common elements with at most 50 subsets, so there exist 50 subsets
PRE",0f
0" 0f") = (e +0f" +af” +af") such that A;, is contained in the complementary of their union. Since each union
pS(an,02,03,04) = Tpm of 50 subsets has more than 2n elements, it follows that each A, has less than
72 Souvrions SoLutions - SeLEcrion Tests 73
Jin clements. But in this case Aj, U A U+ P
U A, has less than $n elements, The polar of B is A'C" and M belongs to A'C", therefore the polar m of M
flse. passes through B. It follows that m is BB'. In the same way the polar of N is
AA'. These show that the pole of MN is P, the intersection of the lines AA’ and
ProBLEM 10. Prove that if n and m are integers, and m is odd, then BB, therefore MN L IP.
2 (oo PROBLEM 12. Let n > 2 be an integer and ay,az, .., @ be real numbers.
Prove that for any non-empty subset § C {1,2,...,1} the following inequality
holds: N
is an integer.
Solution. Let
w = ', Then (Ze)'s X @+ta
= €
I3\
o 3;(;&)“" 1) Solution. Let § = {ir,i1 +1, Jiyingia + Lio.odayeeripeeip) the
ordering of S where ji < ixy1 — 1 for k 12,0 p = 1. Puts, = oy +ag 4o+
S (L4 VBT (L VD (1) ay 50 = 0. Then
The right side of the above equality is the sum of the 3m-th power of the roots
21,22, of the polynomial
= it =s
(X =1 = (3n—1) = X* - 3X% + 3X — 3n,
and
Let sk = 2f + 2§ + 75, Then so= s = 55 =3 and 3 (it tay)? (55
= 51)°.
@ Skes = 35krz — 35ke1 + Bnsk. [P <
It suffices to prove an inequality of the form
1t follows by induction that cach si is an integer divisible by 317, A repeated
application of (2) yields .
Sky7 = 63nsi4a — 9(n® — 3n — B)skyr + 27n(2n+ 1)sk. ) @ =zt ()P < Y (g -m) Yl
1€iGa =
Since s = 9n, it follows inductively that sekss is divisible by 37+ for all non-
negative integers k, and the conclusion follows by (1) because this weans neglecting some nou-negative terms in the right-hand member
of the given inequality.
ProBLEM 11. The incirele of the non-sosceles triangle ABC has center I Tnequality (1) reduces to
and it touches in A, B, C" the sides BC, CA, AB, respectively. The straight lines
AA" and BB' intersect in P, AC and A'C' in M and B'C’ and BC in N, '
Prove that the straight lines IP and MN are perpendicular. Y mm<-nYy
Solution. We will use poles and polars with respect to the given circle. 1< =

This can be obtained by adding up inequalities of the form dryz; < 2(z? + a3),
i< jy j—i = even (for an odd i, z; takes part in [T]
1-1
such inequalities, and
for an even i, z; takes part in H! — 1 of them)
74 Sowumions SOLUTIONS - SELECTION TESTS 75
Fourth Selection Test Solution. By Buler's formula, E = V+F~2, whereE, V and F respectively,
denote the number of edges, vertices and faces of the polyhedron. Let further Vo
and Ey denote the number of vertices and edges belonging to black faces. Since
PROBLEM 13. Let m, m > 2, be an integer. A positive integer n i every vertex from Vo belongs to only one black face, it follows that Vo = Eo
m-good if for every positive integer a, relatively prime to n, one has n|a’ Therefore
Show that any m-good number is at most 4m(2™ ~ 1).
Solution. If m is odd then n|(n — 1)™ — 1 implies n|2, hence n < 2. E-Eo=V-Eg+F-2=V-V+F-2>F-2=n-2,
Take now m = 2'q, t > 1, g odd. If n = 2%(2v + 1) is m-good, then 50 there are at least n — 2 edgos which o not belong to a black face.
(2v+1)[(2v—1)™ — 1, whence (2v+ 1)|2™ — 1. Also, if a = 8v+5 then (a,n) = 1,
24(a7)* 1= (a7~ (a? + 1)(% +1) -+ (@7 4 1) Fifth Selection Test
But a% = 5 (mod 8) implies that the exponent of the factor 2 in the last product
is ¢ +2, therefore u < ¢ +2, whence n < 4-24(2v +1) < 4m(2™ ~ 1), ProBLEM 16, Consider a triangle ABC and O be a point inits interior. The
straight lines 04, OB, OC meet the sides of the triangle in Ay, By, Ci, respectively.
PROBLEM 14. A point O is situated in the triangle’s ABC plane. A circle C Let Ry, Ra, Ry be the radii of the circles (0, B, C), (0,
C, A), (O, A, B) respectively
passing through O meets the second time OA, 0B, OC in P,Q, R, respectively, and and R the radius of the circumcircle of the triangle ABC. Prove that
C intersects the second time the circles (B,0,C),(4,0,C), (4,0, B) in KL, M
respectively. 04,
ol OB
g pRt 4 00
Lip
g s
Prove that PK, QL, RM are concurrent.
Solution. An inversion of center O transforms the circle C into a straight line
d. Points P, @, R become the intersections of d with the straight lines 04, 0B, 0C Solution.
and K, L, M become the common points of d with BC,CA, AB respectively. A
Thus the conclusion reduces to proving that the circles (0, P, K), (0,Q, L),
(0, R, M) have a second common point. In order to do this, it is enough to show
that they have a common radical axis, < B
Let X be the interscction of the radical axis of the circles (0, @, L), (O, R, M)
with d. Then, considering oriented segments,we get XQ - XL = X+ XM and
it is enough to prove that XP- XK = XQ- XL.
By Menelaus theorern for the triangles OPQ, OQR, ORP and straight lines B A c
AB,BC,CA respectively, we get
04 PM
PM QB _, | OB
QB OB 3K RC
@K o L PA_ Notice that
PA QM OB ' QB RK ' RC PL 041 _ area(OBC) _ 0B-0C-BC 4R
A4 T wea(ABC) T T 4Ry AB-AC-BC'
whence
PM - QK- RL = QM -RK - PL.
Take now coordinates on d, with X as origin (denoted with same but small 50 we have to prove that.
leters) Thea (m =)k = )L~ ) = (m = )(k = r)(1 = ) ad g1 = mr, which
leads to (pk — gl)(r + m —g 1) =0. " 0B-0C-BC
> AB-AC - BC.
Ifr+m = g+ 1 we get {M, R} = {Q, L}, in which case the conclusion is
obvious; otherwise pk — gl, whence XP- XK — X XL Consider the complex coordinates 0(0), A(a), B(b), C(c). The conclusion reads
PROBLEM 15. Some of the n faces of a polyhedron are colored in black, in
such a way that any two black faces have no vertex in common. Al other faces SIblfel o —c} > la= b [p~c|-c
are colored in white.
Prove that the number of edges that are common borders of two white faces that is
is at least n— 2. D6 — ] 2 Jab? + be? + ca® — a%h — b — %al,
Sowurions SoLUTIONS - SeLECTION Tests 7
0
which is obvious. Tt follows that /(1) = 0 if and only if the sum of the residues equals the sum of
.
non-residucs. Since Y i+ Y= %, L the latter is possible only if p =
PROBLEM 17. A moue on & m x n board consists of @
(i) Choosing some empty squares such that no two of them ave in the same (wod 4). Tn this case —1 € Q and z & Q implies p — € Q, therefore the »
yow or in the same column and placing a white stone on each of the selected
squares; elements of Q can be grouped in p-1 pairs, each pair having sum p. hence
(i) Placing then a black stone on each empty square which hias a white stone
on his row and on his colum,
‘What is the maximum number of white stones which can appear on the board
after some moves have been made?
Solution. The answer is m +n - 1.
We can obtain m + n — 1 white stones by making succesively the moves g ) We lave alady proved that i thiscase (X 1) s a divsorof /. Notice
(D, (1,2),0- (), (2om) .. () hat
2

Take now a board on which stones have been placed according to the rules,
and remove all the lines and colums which contain less than two white stones;
in this way we remove at most m + n — 1 white stones. If some white stone is
remaining, then on every remaining line and column there are at least two white
stones. Among the remaining white stones, consider one which has been placed
with the last move. Then, with the same move, at least another white stone must
have been placed on the same line or on the same column, which contradicts the We will use the fact that for a prime p of the form 8k +5, we have 2 € N. Indeed
established rules.
2:4:6(p—1)
= (2.4 (4k +2)) ( (4k ) - 8k + 4))
PROBLEM 18. Let p be an odd prime number and ay, i 12 =(2:4 4k +2) (4= 1)+ (-1)) (mod),
be the Legendre symbol of i relative to p (ie. a;
otherwise). Consider the polynomial hence 242, (4k 4 2)! = (~1)?*+1.(4k +2)! (mod p), so 2%+2 = —1 (mod p).
Consider the sets Q< = {r € Q|r < §}, Q> = (r Q| ), Ne = {r €
F=ataX 4ot o X0 Nir<§) No={reN|r>}}.
Since —1 € Q, sending r € R to p—r € R defines a one-to-one function from
) Prove that 1 is a simple root of if and only if p = 3 (tmod 4). Qt0Q, from N to N, form Q< to Q> and from N to N Therefore both Q<
b) Prove that if p = 5 (mod 8), then 1 is a root of order exactly two for f. and @ have £—1 clements and
Solution. We shall call “residue’ and “non-residue’, the elements of the
set R = {1.2,...,p~ 1} which are quadratic residucs mod p and, respectively, =
-1 )72p2r+ >
non-quadratic residues mod p. Let Q be the set of residues and N be the set of rede rede
non-residues. By definition, a; = 1 if i is a residue and a; = —1 if 1 is a non-
residue. Since for 2,y € B the congruence 22 = y? (mod p) is equivalent to = =y
or x4y = p, it follows that Q = {2*|z € R} has
p-1 elements, that s |Q| = [N],
hence £(1) =0 and 1 is divisible by X — 1 [ S D e
a) We have req. reqs
=510, ixwffh In the same way

oL
i@ iew
@) Fol
gy a2y
reNe rene
8 Sowutions Sowutions - SeLEcTION TesTs 7
By the same arguments, sending r to 2r (respectively to p—2r) defines one-to-one Prove that:
maps from Q. to the sets Neyen (respectively, Noaa) of even'(respectively, odd) i) If @ < 5 the sequence contains infinitely many of numbers for which the
uon-residucs. Therefore, sum of their digits (in decimal representation) is not a multiple of 5.
ii) The above result for a = 5.
© Y=Y e-mpeaY A-Eh
v sy e
fek reRe rea req. reds Solution. Consider B = (by) the strictly increasing sequence of all the
nonnegative integers having the sum of their digits a multiple of 5, i.e.:
A similar argument yields
bo=0,by =5,b = 14,bg = 19,5y
= 23,
0] > LACEU
SR Do
= reNe reNe We shall use the following easy result
Lemma. For all positive n we have 5n < b, < 5n + 4.
Combining (1) and (3), then (2) and (4), we get

5res 5 ), Proof. Consider the last digit of the numbers 5, 5n-+1,5n+2, 5n-+3,5n+4
and notice that the union of the numerical intervals [5n, 5n -+ 4] is the set of non-
negative integers.
g E ) Let us now assume that a, takes only a finite number of values with sum
of digits not a multiple of 5, i.e. there exists an integer M > 0 such that n >
M implies @, € B. But then, for any integer N > 0, consider the numbers
MM +1,...,M +N. Since aryx € B for k = 0,1,2,...,N, and because a,
takes each value at most once, we have
®)
SN <y < max o < max oM +) = a(M + N),
Using (1) and (4) we get
o absurd for a < 5, as N is arbitrarily large.
aM
2ol
Ple-1) 44 5,
ivE 5, take ag = 1 and ay = by, for n > 0; we have by
n — 1 < 5n, 50 a,, < 5n for n >0 but a, € B.
< 5(n —1) +

=Bzl 4,5, PROBLEM 20. Given an integer number n > 1, consider n distinct unit
ren vectors in the plane, which have common origin at some point O. Suppose
which leads to 3~ r= 5= 7. On the other hand further that for some non-negative integer m < 3, on cither side of any straight
& o iR line passing through O lic at least m of these veetors. Prove that the length of the
sum of all n vectors cannot exceed n — 2m.
Treyeey
cEReENe <
=odd, Solution. Throughout the proof vectors are written in bold-face characters.
Let v denote the sum of all 7 vectors. Leaving aside the trivial case v = 0, assume
henceforth v # 0 and consider a standard orthogonal frame 20y whose positive
a contradiction. This shows that the assumption in (5) is false. a-axis s directed along v. The plane punctured at O comes out with a natural
partition into four quadrants :
Supplementary Test @ ={(zy)|z>0andy >0},
PRrOBLEM 19 Consider a sequence of positive different integers (ap)n such (z,y)|z<0andy >0},
that there is a > 0 satisfying Qi ={(®y)|z<0andy<0},
an < an, for all positive integers 7. (z,y) |z > 0and y < 0} .
80 Sowumions
Let further vi, . , Vi be an anti-clockwise sequential order of the given vectors.
Thus, for some integers p, g and r, 0< p < g <r <,
&) the v, lie in @, for 1< < p, and the order is vy, . , v §
b) the v; lie in Q for p < i < g, and the order is Vpe1, . y vy 5
©) the v lie in Qg for g < < 7, and the order is oy, . , vy
d) the v; Tie in Qu for r < i <, and the order is Ve, ..., vr -
The cases p = 0, or p = g, 0r = 7, 01 1 = n are not necessarily exclude 2.4. THE NATIONAL MATHEMATICAL OLYMPIAD
they simply mean that 1o v; lies in Q1 or in Qa, or in Qs, or in Q, respectively.
Applied to the lines = = 0 and y = 0, the condition that on either side lic at least Selection Tests for the JBMO 2004
m of the v; yields
&) mep<r, and mir<ntp,
o the one hand, and PROBLEM 1. Find all positive real numbers a,b,¢ which satisfy the inequal-
ities
2 m<q, and mtgq<n,
4ab+be+ca) — 13 a® + B 4 & 2 3(a®
+ 57 4+ ),
on the other. By (2) above, the vectors
Solution. Using the Chebyshev inequality we derive
Voemii Ve, 1<i<m,
(a4 b+e)a® + b +c?) < B(a® + b° + %),
are well defined. We claim that they always pont at the left half-plane z < 0
that is, they have a non-positive 2-component hience a+ b+ ¢ < 1. On the other hand,
® Ve (Ve Vo) S0, 1<i<m A(ab+be+ca)—1 > a® + b + ¥ > ab + be + ca,
To prove (3), fix an index i, 1 < i < m, and note that Vs always lies in therefore ab + be+ ca > 1. As
Q1UQs, and vquy always lies in Qg U Qu. Clearly, only the cases where Vq-m i
lies in Q1 or v lies in Qg need to be dealt with. B(ab+be+ca) < (a4
b4 o) <1,
In' the first case, g —m+i < p, 50 g+i < m+p < r by (1); that is,
Vys: must lie in Qs. Recalling the way the veetors have been ordered and that we obtain a+ b+ c > 1, thus a+ b+ 1. Consequently, a +b+¢ = 1 and
at least m of them lie on either side of the line along Ve, we deduce that 3(ab+be+ ca) = (a+ b+ c)?, which imply
Vsi = Vig-miyem cannot lie outside the angle (v, —Vy_p 1) situated in Qs
and (3) follows.
In the second case, 7 +1 € g+, 50 p+1 <7 —m+1<q—miiby
(1); that is, Vo-pss must lie in Qo As in the first case, we then deduce that
Vg-mi = Vigai)-m cannot lie outside the angle £(~v, ~vq.s) situated in Qz and
(3) follows again PROBLEM 2. Consider the numbers defined by a, = 3n + vn? — 1 and
Finally, since the length of the sum of the remaining n — 2m unit vectors by =2(vnT+ n+nZ—n), for all n = 1,2,...,49. Prove that there are integers
cannot exceed n — 2m, we conelude by (3) that neither can the length of v. A, B so that

Vo1 = b1+ Vg~ byt e+ +BV3.

Solution. The key idea is to observe that

an—ba=t 2 (WATI-2va+vaTl)
82 Sowvmions SoLurions - JBMO SeLgction Tests 8
As 2@ 2 VA~ 1+ va—1, it follows that the sum is 4v/2- 5. a contradiction,
2) MN s a diagonal of the cube. Selecting the vertices M, then N, and
ProBLEM 3. Consider a circle of center O and V a point externally to the performing these 2 movements, to all the vertices the tumber 1 will be assigned,
circle. The tangents from V' touch the circle at points Ty,Ty Let T be a point as needed.
on the small are T,T of the circle. The tangent at T intersects the line VT} in 3) MN is a side of the cube. The same outcome as in the previous case
the point A and the lines T; and VT, intersect in the point B. Let M be the will oceur after 2 movements when selecting the diagonally opposite vertices of
intersection point of the lines BT} and AT, M and N.
Prove that lines OM and AB are perpendicular. This provides us with a full solution.
Solution. The approach of the problem is to see no circles in the figure.
Instead, recall that a quadrilateral ABCD is orthogonal if and only if PROBLEM 5. Let ABC be an acute triangle and let D be a point; on the side
BC. Points E and F are the projections of the point D on the sides AB and AC,
AB® 4 CD* = AD* + BC?. respectively. Lines BF and CE meet at point P. Prove that AD is the bisector
line of the angle BAC if and only if lines AP and BC are perpendicular.
Solution. Let a,b,c, .y be the lengths of the sides BC, CA, AB, BD, DC,
respectively and let A’ be the foot of the altitude from A in the triangle ABC.
Notice that 7+ y = a.

>
A

Using succesively the Pytagoras theorem we have


B
AN
D c
BA? - BT} = BA® — (BO* - OT}) = BA® - (BO? - OT})
Due to the Ceva theorem, the claim is equivalent to
= BA? — (BA® - AT?) = AT? = AO® - OT} = 0A* - OT},
i AB BD
CF AE
pE =L whichis cquivalent to 55 = 52
BD
50 the conclusion follows from the relation. DG FA
PROBLEM 4. Consider a cube and let M, N be two of its vertices. Assign c0sC,FA =b-ycosC,BE = xcos
B, AE = c—xcos
B, BA' = ccos B
the number 1 to these vertices and 0 to the other six vertices. We are allowed to 05C, the equivalence rewrites
select a vertex and to increase with a unit the numbers written in the 3 adjiacent
vertices and call this 2 movement, Prove that there is a sequence of movements eyle— zcos B) = ba(b ~ yeosC),
after that all the mumbers assigned to all the vertices of the cube are equal, if and
only if MN is not a diagonal of a face of the cube. which is the same as zb = cy. Indeed, we have
Solution. Color the 8 vertices of the cube in black or white so that the
4 vertices of the 2 regular thetrahedrons have the same color; notice that the 3 el2 -1 I a4 o -
P st
vertices adjiacent o a vertex have its opposite color. Therefore, each movement YT e R
inerease the sum of the numbers assingned to the vertices sharing the same color
by 3. Consider the cases: equivalent to a(ely b2) = zy(c? 1), or y(a — ) = B(a - y). This gives
1) MN is a diagonal ofa face of the cube. Then M and N have the same y? =142, equivalent to cy = b, as claimed.
color, say black. Assume by contradiction that there s a sequence of movements
after which the same number n is assigned to all the vertices. Let ki and kp be PROBLEM 6. An array 8 x 8 consists of 64 unit squares. Inside cach square
the number of white, respectively black vertices that were selected to perform the are written the numbers 1 or ~1 50 that in any 2 x 2 subarray the sum of the four
‘movements. Then numbers equals 2 or —2. Prove that there exist two rows in the array which the
An =3k +2 =3k, seme numbers are inseribed in the same order.
81 SoLuTions Soturions - JBMO Sevecrioy Tests 85
Solution. The main idea is to observe that two consecutive rows have We have
exactly 4 equal clements, namely those lying on the columus 1,3,5,7 or 2,4,6,8 .y
Moreover, on the other 4 columns the clements are different. Without loss of 1
)y -n
generality, assume that rows 1 and 2 are equal with respect to the columns 1,3,5, 7 n »xk
and different on the column 2,4,6,8; we call these rows odd equal. If ows 2 and
3 are also odd equal, then rows 1 and 3 are equal, as needed. If not, then
and 3 are cven equal. Now consider the rows 3 and 4; we are done if the rowsrowsare2 :H(nq) (fl+l)2k
even equal, so assume that they are odd equal. Finnaly, if rows 4 and 5 are odd
equal then rows 3 and 5 are equal, and if rows 4 and 5 are even equal, then rows
Land 5 are equal. This concludes the proof. For n > 6 we prove by induction on n that
ProBLEM 7. Consider a triangle ABC with the side lenghts a,b, ¢so that a 1
is the greatest. Prove that the triangle is rightangled if and only if
(\/n+b+\/ofb> (VAT e+ va—e) = (a+b+c)v/a. yhich implis tht the given equlity cannot hold. Tndeed, for n — 7 we bave
T=1T53 4444 . If the inequality holds for n > 7 then it is
Solution. Squaring both sides of the equality ields true for n+1,as 1 > 2Ly
We are left, with the cases when 7 = 2,3,4,5,6. Clearly, the case case n = 2
(o4 VA=)
2(a+Va? a+ (o @) =(a+b+cp. 3 we have the mumbers a; 2 and ag son=3isa
solution.
It is easy to observe that the equality holds if a? = b + 2. To prove the converse If n =4, then
statement, assume that a® > b + 2. Then va? — 7 > c and va? = > b, hence
2(a+ Ve V) (a+VaT= &) > 20+b (a+¢)= 2 + 2(ab + be + ca) @ tartatag P
>a? 452 + ¢ + 2(ab+ be+ ca) = (a4 b+ c)?,
50 a3 +az+a3 +a4 < 12. By inspection, all the cases: {a1,a2,03, a1} = {1,2,3,4},
{2 z 5}, {1,2,4,5) and {1,2,3,6} fail to satisfy the required relation,
false. The case a? < b2 + c? leads similarily to a contradiction, and we are done. f n =5, then
PropLew 8. Find all postive integers n for which there are distinct integor x2345)
numbers ar,az, @, such that
o e aa as
1,2, n_mtatote
ol a
Solution. Rearrange the numbers 1,0z, -, a, in ascending order: b, < 50.a1-+a2-+as+as-+ag < 17. We study the cases {a1, 02,09, 04,05} = {1,2,3,4,5},
by <. < by. Obviously, k < by, and substituting by with F, the left-hand side {1,2,3,4,6} and {1,2,3,5,6} with no succes (for an easy argument, observe that
term increases. Futhermore, by the rearrangements inequality we infer that the 5 must be as and so on)
maximum value of the loft-hand side term is Finally, for1 = 6 we hae a1 ag-+-- o < 22 thus {a,aa. 01,500}
can be {1,2,3,4,5,6} or {1,2,3,4,5,7). The last case fails immediately because
of 7, and the same outcame is for the first, one.
Therefore 7
On the other side, the right-hand side term is greater than or equal to PROBLEM 9. Tn a chess tournament each of the players have played with all
1424 the others two games, one time with the white and then with the black. In each
game the winners gets one point and both players receive 0.5 points if the game
86 Sowvtions Sowurions - JBMO SeLscrion Tesrs 87
ends with a tie. At the end of the tournament, all the players end with the same Prove that n =
number of points Solution. Clearly one of the primm p.g or v is equal
a) Prove that there are two players which have the same mumber of tics. to 2. If r = 2 then
b) Prove that there are two players which have the same number of defeates g = d, false, 50 assume that p > g = 2.
when playing the white. Consider the case when n > 1 is odd; we have
Solution. Let n be the number of players in the tournament. The total (PH+2)(p"" = 25" 4 27— 27 =2,
numbers of matches is n(n — 1), hence each player end up with n — 1 points,
a) Assume by contradiction that each player has a different number of ties. Notice that p"~3 —2p"~24 22512.. 4311 = 301 (o) (P gt o) > 1
As a tie gives 0.5 points, it follows that each player has an odd number of tics. Since and p+2 > 1 hence both factors are equal to . This rewrites
the possible cases are: 0,2,4, ..,2(n~ 1), we infer that each of these numbers is as p" 42" = (p+2)% =
assigned to each of the players. Consider A the player with 0 ties and B the player P*+4p + 4, which is false for n > 3.
with 2(n ~ 1) ties. Bach player has played 2(n — 1) matches, hence B obtained a Consider the case when n > 1 is even and let n = 2m. It follows that =
tie in each match played. The match A-B thus ended with a tie, a contradiction, a?—b%,2™ = 2ab and r = a? + b2, for some integers a, b with (a,b) = 1. Therefore,
since A has 1o ties. aand b are powers of 2, s0 b= 1 and
50 p must be equal to 8. The equality
21 This implies p™ = 4™~ —1 < 4™,
b) Suppose the contrary. Then each of the n players has0,1,....,n — 1 losses =1~ 1 fails for m = 1 and also for
when playing the white. Let X and Y be the players with 0'and n — 1 losses, m 22, as 4™1 > 3™ 4 1, by induction.
respectively. The player Y has no points when playing the white and n —1 points, Consequently 7 = 1 take for example p = 23, g = 2 and r
50 he won all the matches with the black pieces. This implies that the match X-Y
is won by Y, so is lost by X, & contradiction, since X has 0 losses with the white PROBLEM 12. Let a < b < ¢ < d be positive integers so that ad = be and
pieces. Vd—/a < 1. Prove that a is a perfect square.
PROBLEM 10. Consider the triangle ABC with AB = AC and a variable Solution. Consider the integers 0 < m < 1 < pso that b= a+n,c=a+m
point M on the line BC so that B is between M and C. Prove that the sum of andd=a+tThen p afa+p) = (e+m)a+tn) anda+p < a+1+2/a As
the inradius of AMB and the exradius of AMC corresponding to the angle M is P=m+nt %2 is an integer, then a < mn and p > m+n+ 1. On the other
constant, hand, 1+2y/a > n+m+1= v/a > 2§ > /i, hence a > mn. Consequently,
Solution. The Stewart relation gives AM?. BC+AC?- MB = AB*- MC +
a=mn,m=mn, soais asquare.
MB - MC - BC, so AM? - BC = AB*MC ~ MB) + MB - MC - BC, hence PROBLEM 13. Lot ABC be a triangle inscribed in the circle K and consider
AM? = AB* + MB - MC. Let r be the inradius of triangle AMB and R the a point M on the arc BC which does not contain A. The tangents from 1 to
exradius of triangle AMC corresponding to the angle M. the incircle of ABC intersect the circle K at the points N and P. Prove that if
2arca AMB area AMC
Since 1 ~AM+MBy AB wd Ry MB- B then LBAC = (NMP, then triangles ABC and MNP are congruent.
r MB R Solution.
B AT MBTAB ™ % T AT e 4B Thus r + R = h is equiv-
MB(MA+ MB— AB) + MC(MA+ MB + AB)
= (MA+MB 4 AB)(MA + MB - AB),
which turns to be equivalent to MB(MA+MB—AB) = (MA+MB+AB)(MA—
AB), or to MB- MC + MB(MA ~ AB) = MA? ~ AB? + MB(MA~ AB). The
last cquality reduces to
3
MA? = AB® + MB- MC, Let @ be the intersection point of the line segments AB and MP. The
50 the claim holds. tangents from A and M to the incirle are equal (as they are r - cot4). More-
over, the tangents from Q to the incircle are equal, so AQ = MQ. This implies
PROBLEM 11. Let p,q,r be primes and let n be a positive integer such that LQMA = LQAM, so the arcs AP and BM are equal. In the trapezoid APBM,
the d[iagonals AB and MP are equal, and likewise AC = MN. This concludes the
P proof.
88 Sowvmions Sowurions - JBMO SaLecrion Tests 80
PROBLEM 14. The real numbers ay,az, .., ay satisfies the relation Proof. Form two pairs of consecutive vertices and change them in the same
color - if they do not already have it. Then follow the sequence 1122 — 1002 —
@+l (ot o)= 101 2202 — 2112 — 0000,
Prove that [ag| < 10, for all k =1,2,...,100. By the second lemma, after several moves the vertices Ay, Az, s, Ag will
have the same color, say red. Likewise, As, A, A7, As will have the same color.
Solution. Assume by contradiction that |ax| > 10, for some k. Wlog, let Consider now the vertices Ay, As, Ag, Ar; the first is red and the other three have
k= 1. Theno > 100 and a3+a3+...+ao0+s2 < 1, where s = a1 +az+.. . +a100. the same color. By the first lemma they all will turn red - of course, we do nothing
On the other hand, the Cauchy-Sclwarz inequality yields if they were already red. We move on with this procedure until Ay, Aa, .. Agar
turn red (note that 997 = 4 +3-332, so this requires 332 steps). Now consider the
af = (s —az 03+ ... — aioo)? < 100(aF
+ a3 + ... +adoo
+ %) < 100, vertices Apgs, Asgo, Atooo,A1; by the second lemuma they all will share the same
color. I this is red, we are dove. If not, say that they are blue, and taking the
a contradiction. vertices Asg, Asns, Aswa, Aroco We obtain- using the first lemma — all vertices to be
red, except for Ay, which is blue. Now Ay, A, Ay, Ag turn blue, then As, Ag, A7, Ag
ProsLEM 15. A finite set of positive integers is called isolated if the sum and 0 on. This time, after 333 steps, all the 1000 vertices (1000=1+3-333) will
of the clements in any proper subset is a number relatively prime with the sum be colored in blue.
of the elements of the isolated set. Find all non-prime integers n for which there Comment. Substituting colors with digits, notice that all moves: 01—22,
exist positive integers a,b so that the set A = {(a +b)%, (a + 262, . (a + nb)?} 0211 and 1200 preserve the sum (mod 3). This means that the fnal color
s isolated. s unique and, of course, is given by the sum of the digits assigned to the vertices
Solution. of the initial configuration.
The sum of the elements of the set A is § = na? +n(n+1)b+ 220 g2
Assume that n has a prime divisor p > 3. Then p|S and p(a-+b)?+ (a+28)+-- -+ PROBLEM 17. Consider the triangular array
(a+pb)? = pa? + p(p-+ 1)b+ B, 4 contradiction. Tt remains n = 2¢3',
for some integers k. L. Suppose that k > 1. Then 2|3 and so all elements of 4 must 01123 5 8
be odd. Taking the subset given by any pair we reach the contradiction. Finally, 011 2 3 5
suppose that { > 1, 50 3[S. If one of the numbers a-b,a-+2b, a-+3b s divisible by 3, 235 8 13
then we have a contradiction; if not, then 3[b. Then 3|(a-+b)*+(a-+2b)?+(a+38)* = 47 1118
3a® + 12ab + 1462, again a contradiction. 12 1931
W are left with 7. = 6. It satisfies the claim: the set A = {4,9,16,25,36,49} defined as follows:
is isolated, because the sum of its clements is a prime number (139) ) on the first two rows, each element starting with the third s the sum of
Prosuem 16. A regular polygon with 1000 sides has its vertices colored the two preceding elements;
in red, yellow or blue. A moue consists in choosing to adjiacent vertices colored ii) on the other rows each element is the sum of the two clements placed
differently and coloring them in the third color. Prove that there is a sequence of above of the same colunn.
‘moues after which all the vertices of the polygon will have the same color. a) Prove that all the rows are defined according to condition i).
b) Consider 4 consecutive rows and let a,b,c,d be the first element in each
Solution. Let Ay, As, .. Ao be the vertices of the polygon. We start with of these rows, respectively. Find d in terms of a,b and c.
two lemmas. Solution. a) Tn the array below
Lemma 1. Three of four consecutive vertices have the same color. Then after
a sequence of moves all vertices will have the color of the fourth vertex.
Proof. Let the colors be 0,1 and 2. We have two cascs:
©as

SR

a) 1110 — 1122 — 1002 — 2202 — 2112 — 0000,


b) 1011 — 1221 — 0000. where i is an element of the third row, observe that i = ¢+ f = (a-+b) + (d-+¢)
Lemma 2. Any 4 consecutive vertices will turn after several moves in the (a+d)+ (b+¢) = g+ h. The same argument holds for all the other rows, by
same color. induction.
9 Soumoxs SoLuTIoNs - JBMO Sercrion Tests 91
b) We prove that d = 2b + 2¢ — a. Indeed, from the array PRoBLENM20, Consider convex polygon with n 3 5 sides. Prov that there
are at most n(2n
"= =5 tangles of aren 1 with whose vertices are choosen from
the vertices of thé polygon.
Solution. Let Ay, Az, Ay be the vertices of the polygon. We start with
the following
we derive
d = u4v = bkt 4 (ut2) =2(b+1)
+ (+Y) B2t
+y) Ty = Lemma: Each segment 4;A; belongs to at most 2 triangle of area 1 located
2b+2c+T— 242 —a. on the same side of the line 4,4,
Proof of the lemma. Indeed, suppose that on the same side of the line A;4;
ProBLEM 18. Let M, N, P be the midpoints of the sides BC, CA, AB of the exist the vertices Am, Ay, A 50 that the triangles AiA; Ay, AiA, Ay and A,A;4,
triangle ABC, respectively, and let G be the centroid of the triangle. Prove that have the area 1. Then the points An, A, A, will be at the same distance to the
if BMGP is cyclic and 2BN = V3AB , then triangle ABC is equilateral. Tine A;4;, hence colinear. This is a contradiction, since the polygon is conves.
Solution. By the power of a point theorem we have AG - AM = AP - AB, Consider first the n sides of the polygon. Each of them can form at most 2
50 4MA2 = 3AB? and thus AM = ¥2AB = BN. triangles of area 1, as all the vertices lie on the same side, hence we have by now
at most 2n such triangles.
4, Consider now the n diagonals A, A4 - with the eyclic notations: A, = A;
Bach of them can form at most 3 triangles of arca 1, one with Ay and two with
the vertices lying on the other side. Thus we have af most 5n = 2n 4 3n triangles.
Finally, consider the other diagonals of the polygon. They are 2% and
each of them can form at most 4 triangles. The final counting is 5n + 42052 —
(20 — 5), except that we have counted each triangle three times, one time for
B I3 c each side. Therefore, there are at most “23=9 triangles, as claimed.
Then AG = GB, so the median GP s also an altitude in the triangle AGB.
This implies ZBPG = 90°, and since BMGP s cyclic, ZGMA = 90°. It follows
that BC = CA and AB = AC, so the triangle is equilateral.
ProbLEM 19. Let A be a set of positive integers with the properties:
) if a € A, then all positive divisors of are elements of A;
i) if a,b€ A and 1 <a < b, then 1 +abeA
Prové that if the set A has at least 3 elemonts, then 4 = N".
Solution. It is obvious that 1 € A, since 1 is a divisor of any integer
Consider a,b two elements of A with 1 < a < b. Since at least one of a, b or 1-+ab
is even, then 2 is an element of A.
We induct on n > 6 to prove that n & A. Assume that k € A for all
F=1,2...,n— 1L Ifnisodd, thenn = 2+ 1 with 1 < 2 < p € 4, hence
€ 416 n is even, then n = 3p. As above, 2p ~ 1 and 2p + 1 axe elements of
A and consequently 1+ (2p — 1)(2p + 1) = 4p* € A. The first property implies
=2p € A, a3 needed.
To complete the proof, we need to show that 3,4,5 € A. For this, consider
4> 2 an clement of A. Then 1+2-a € A, 1+42(1+20) = 3+4da € 4 and
15 (1420)(3 +4a) = 4+ 100 + 82 € A. If a is even, then 4|4+ 10a + 8a?
and 04 € A. If a is odd, then choose a to be 4+ 10a + 8a? and again 4 € A
Next, a8 1 <2<4€ Awehave 1+2:4 =9 € 4 and 503 € A. Finally,
T=1+2-3€ A15=1+2-7¢€ A hence 5 € A and we are dore.
SoLUTIONS - REGIONAL CONTESTS 93
Observe that
a?? —a% —ab? + ab=abla—1)(b—1) >4
and |
a®+b'—a—b—ab 7((a7b)2—\ afa—2)+b(b—2)) >0.

2.5. REGIONAL MATHEMATICAL COMPETITIONS The inequality becomes an equality for a


3. Let ABC be a triangle, M be the foot of the altitude from C and N be
Problems and Solutions the refiection of M across the line BC. The parallel line to CM through the point
N intersects BC in P and AC in Q
a) Show that MQ L AP if and only if AB = AC.
b) Show that it is possible to obtain the points A, B,C when the points
M,N, P are given
Solution. a) We have AB L PQ. Therefore MQ L AP if and only if M is
the orthocenter of the triangle APQ, that is if and only if MP L AC. Because
LMPC = LNPC = (MCP, it follows that PM L AC'if and ouly if £B = /C.
7" GRADE b) In the given conditions, B is the oxthocenter of the triangle MNP, C
is the intersection of the altitude from P with the parallel line through M to
1. Let m,n be positive integers. Show that 25" — 77 is divisible by 3 and PN and A is the intersection of MB with the perpendicular bisector of the
find the least positive integer of the form [25" — 7 — 3™, when m, n run over the segment BC.
set of non-negative integers.
Solution. ce 25 (1od 3) and 1 (mod 3), it follows that 257 7™ 9" GRADE
o (mod 3).
For the second part of the problem, we first remark that if m is odd, then 1. Let a,b,c be real numbers. Show that
any number a = 25" — 7™ — 3™ is divisible by 15. This follows from the first part
toghether with
TM 3™ = 27 4 ()™ =0 (mod 5). Va+Vo+ o= VYarbie
Morcover, for m = n = 1 one obtains 25 — 7 — 3 = 15. if and only if
Assume now that m is even, say m = 2%. Then @4 = (@ +b+o)’
T™ 37 = 7 4 3% = ((=3)2 4 3%%) (mod 10) Solution. Using the well-known identity
=2-9* (mod 10) = 2 (mod 10) = 2 or 8 (mod 10),
(a+b+e)=a®+
5+ +3(a+b)(b+c)c+a),
o, the last digit of the mumiber 257 7™ —3™ i ciher 3 or 7. Bocauso the mumber we see that (a-+b-+ c) 46+ ¢* if and only if either a+b =0, or b+ =0,
51 -7 37 is divisible by 3, the requived number cannot be 7. The situation
\M' 737 3 can b ok oecun, becmuse 26 ~ 7 3% = 1 (o 8). oreta=0. It follows that
2. et a,b be real numbers such that Ja] > 2, [5] > 2. Show that Va+ Vbt o= Varbre

(a® + 1) 1) > (e b)(ab 1 1) 1 5, For the converse, one may apply the same argument for the numbers /a V5, J2.
and find when equality holds. 2. Let 7,9,z be real numbers such that
Solution. We have P hyz<2 P ez A ray<2
(a?+1)(6*+1) — (ab+1)(a—b)5 = (a**—a%b - ab>+ab
+ (a® +b2 ) —a—b—ab) Find the minimal and the maximal value of the sum = + y + 2.
94 Sowmioxs SoLuTions - RecioNaL CovTests 9
Solution. Adding up the given inequalities, one obtains Solution.
@y 2 hmy+yz
4w <6,
Taking into account that oy +yz-+22 < 22 447+2% one obtains 2(a-+y-+2)? < 18.
Therefore, ~3 <z +y + = < 3.
The values 2=y = > =1 and ~1, respectively, show that both
lower and upper bounds are attainable.
3. Wo ate given the set A = {1,3,6,10,15,2 }. Show that there exist
numbers ar,az, .., azoos € A such that, Let P,Q be the intersection point of EN, KM and FK, NM xespectively.
‘The triangles N PM and KQM are right angled, satisfy LN MP = LK MQ, there-
@1 +az 483+ - + azong
= azoou, fore they are similar. It follows that ZPNM = MKQ and also LENM =
LFKM.
Solution.The set A s the set of the so-called triangular mumbers:
5. Let n be a positive integer and a, b, ¢ be real numbers such that a” = a-+b,
A={Te=14243++k|k>1}. b =b+cand
Show that
We will prove by induction the following statement: for any k, & 3 2, there exist Solution. If n = 1 the conclusion is obvious. let n > 2. We have a”
triangular numbers T3,,T, . , Ty, ,, such that a=cb" —¢" =b—a,c" —a" =~ b, Distinguish two cases:
Case 1. n is odd. Then the function f(z) = 2" is monotonic increasing
T+ T+ + Ty =Ty, on the real axis. If a > b, one obtains a > ¢,c < b and then a < b, which is a
contradiction.
For k =2, we have Ty + Ty = Ty, Assume that Case 2. is even. If a,b,c > 0 the conclusion follows like above. Since
a+b3>0b+c30,c+a >0, we cannot have negative numbers. Assume that
Ty 4Tyt 4T, = a<0andb 0,63 0. Leta ==,z >0. Thena" = —z+b,b" = b+c,c" = c-x.
It follows b > 7,¢ > 2. Since " — " = b c we get b < c. On the other hand
Since T, = EEHD et b~ " = btz >0, that is b > c, a contradiction.
6. Let ABCD be a convex quadrilateral and M, ,P,Q be points on the
Togen_, =142+ +( sides AB, BC,CD, DA respectively, such that

Then M4 _NB_PD_
MB~NC~
04
PC” QD
+ Ty + Tapen
where k # 1. Show that S(ABCD) = 2S(MNPQ) if and ouly i S(ABD) =
4. The circles C; and Cy intersect in distinct points S(BCD)
through A intersects again Cy in C and Cy in D and let beA, anB. arbitrary
An arbitrary line Solution.
the segment CD. The parallel line to BC: threngh M Minteracto point on
in K and the parallel to BD through M intexsects the segment theBC scgment 5D
perpendicular in N to BC intersects the arc BC of C; which does not incontain
N. The
in the point E. The perpendicular to BD in K intersects the are BD of C, whichA
does not contain A in F.
Show that ZEMF = 90°.
9% SoLuTions
SoLuTioNs ~ REGIONAL ConTesTs o7
AM_MQ K PD
AB T BD Solution. Put 3/[7] = a,. Then [z"] = a7 and a} < o™ < af + 1. Taking
toots, one obtains a, < < /a7 1. This shows that [z] = .
We will show that all positive ntegers z, x > 2, satisfy the condition. As-
sume, by way of contradiction, that there s a solution which is not a nonnegative
=1, implying BD = MQ+ PN. integer. Putz=a-+a,a>1,0<a <1
Let by,bz be the lmgth ofthe perpendiculas from 4, respectivel, to BD It follows that a" < (a +a)* < a” +1, and therefore,
and wv the length of the perpendiculars from M, N respectively, to BD. Since
the tisngle AMGQ is similar to vith ABD, and CPN is similas to DB, we et 142aye i=<
<(+8) 1
H * and 1 ey mdv=
Since KINPQ it waperotd On the other hand, by Bernoull inequality,
Loy + MQ) (
S(MNPQ) = (PN Z(k -
])(u; +khy). (1+2)"
a 21402 a 52,
for suffciently large n, a contradiction.
Tt is clear that S(ABCD) = B2 (i + o).
Therefore, S(ABCD) 2S(MNPQ) which s equivalent to
(k4 1)(hy + ha) = 2{0i2 + k), 05 10 hy = o, that s the same with S(4BD) = 10 GRADE
S(BCD).
1. Find all arithmetic sequences
n1, n, i3, ng, ns, for which 5ny, 2(na, 1jna,
7. Let ABC be a right triangle such that, /A = 90°, /B > /C, and let 7lny and 17)ns,
D be an exbitvary point on the segment BC. The angle bisectors of ZADB and 2004, Focsani Contest
£ADC intersect the sides AB and AC in the points M and N, respectively. Show
that the angle between the lines BC and MN is §(B — C) if and only if D is the
foot of the altitude from A, Solution. We take ny = 5a,nz = 2b,n5 le,ng = 7d and ns = 17e. We
Solution. Let P be the intersection point of BC and MN. Since ZMDN =
requite the conditions: ny +ng = 2na,na +ny 2n3,n3 + 15 = 2ng. From these,
90°, AMDN is a cyclic quadrilateral. It follows that
we derive the system of linear equations:
{511. —ab +lle =0
LAMN = £ADN = and ZANM #q. 2 -2 +7d
le —1dd 417¢
‘W solve it in rational numbers by expanding o, b, in terms of d, e, and oe obtains
50
M, (]

P B D c
From these, one has: d = 2z, 6z — Ge 0 (mod 11) and = ~ ¢ = 0 (mod 5.
Therefore z — e = 0 (mod 55), ¢ — 55y, where 2, € Z. Thus
We have [MPB = LABC — LAMN. Hence, LMPB — }(B - O) is
equivalent o {ABC = LADC ~ /BCD, which is the same as 90° ~ (BCD = €+ 561y, b=4z+ 935y, ¢ +85y, d=2q, e=1z—55y.
LADC ~ LBCD, or LADC = 90°.
It follows that d is even, d = 2z, and
8. Find all real numbers7, 7 > 1, such that {/[z"] is an integer number for
all positive integers n, n > 2. 287~ 17¢
1
9 SoLuTios SovuTions - RecionaL Coxresrs 9
The required sequence is Wee obtaina_ £ VBThis shows that OA and OB are perpendicular and
=i
=5z + 2805y
ny =8z + 1870y
112+ 935y o[2) —tan(0B4) = X2V3
ng=1lde Therefore ABCD is a thombus and £B = £D = 60° and £A = £C = 120°.
172 - 935y,
Alternative solution. This is an essentially metric argument. Observe
‘We remark that the ratio of the sequence is 3z — 935y. that one can easily check that in the triangle ABC the medians AN and CM are
equal. This implies that the triangle is isosceles (using, for example, the median
2. Let ABCD be a convex quadrilateral and M, N, P, Q be the midpoints of formula). Tn the same way the triangle DAC is isosceles. The median formula
the sides AB, BC,CD, DA respectively. imply that the two triangles are equal, that is ABCD is a thombus. Checking the
Show that if ANP and CMQ are equilateral triangles then ABCD is a angles is now an easy task.
rhombus. Find the angles of ABCD.
Solution. Let O be the intersection point of the diagonals AC and BD. 3. Let A = {1,2,3,4,5). Find the number of functions f : A — A, with
Since ABCD is a convex quadrilateral, O is an interior point.
the following property: there is 1o triple of distinct elements a, b,c € A such that
fla) = £(b) = £(e)-
Solution. There are three types of functions which do not posses the given
property:
1) Functions which assign to three distinct clements a,b,¢ an clement a’ € A,
and to the remaining two elements, arbitrary elements from A\ {a'}. The number
tions iis ()5 5.4,-5+
of such funetions
2) Functions which assign to four distinct elements from A an element a' € A
and to the remaining fifth element in A some element from A\ {a’}. The number
We shall first give a solution using complex coordinates. Let a,b,c,d be the of such functions is (°) -5 4.
complex coordinates of A, B,C,D respectively, such that O is the origin of the 3) Constant functions; their number is 5.
complex plave. Then M, N, P, Q have complex coordinates atb o bte ‘The required number is thus, 5° — ((3) 542 — (§) -5-4 - 5) = 2220.
7 "T T
d d+a -
p= 5% 0= "5 respectively. The condition that ANP and CMQ are equi- 4. Let a > 2 be a an integer. Consider the set
Interal triangles gives us the cqualities
A= )
2+ (a+b)e+(d+a)e?=0
) Show that A does not contain an infinite geometric ratio.
® 2a+ (c+d)e+ (b+e)e? =0, b) Show that for any n.>> 3, A contains n numbers which are in a geometric
ratio.
where € = cos% +isinZ. Taking into account that €2+ + 1 0, the above Solution. a) Observe that a C [1,y/a]. Assume that (za)y is an infinite
equation yields geometric ratio of ratio ¢. Then 1 < 71¢™ < v/, for all n > 1. In the case ¢ < 1
-d 2a-b the inequality 1< 21" gives a contradiction and in the case g > 1, the inequality
q"7) < /a gives a contradiction.
It follows that ¢ +a = b +d. As AC and BD intersect at O, this gives a+c = 3
b) The numbers a¥',a#,...,a% are in geometric ratio.
b-+d =0. Therefore ABCD is a parallelogram and ¢ = —a,d = b,
The last equalities, when introduced in (1), give 5. Let ABCD be a tetrahedron such that the medians starting from vertex
Ain the triangles ABC, ABD, ACD aze mutually perpendicular. Show that all
2a+(—a—be+(b—a)e?
=0, or 2a—ae—be+(b—a)(-1-¢)=0. edges that contain A are equal.
100 SoLuTions S0LUTIONS ~ REGIONAL CONTESTS 101
Solution.We shall use vectors. Put AB = b, AC = ¢, AD = d. Hence, the
medians are given by the vectors 4(5+c), 3(c +d), }(d+5). The scalar products
of each pair is zero. We thus obtain
b+ (et ed o db) = 0
&+ (be+ed + db) g
&t (be+ed +db) = 0, AR B
which easily gives b = ¢ = d2, that is [b] = =1d =0. The cquation of the line BD s © + ¥ = 1. Therefore, we have
6. Let x,y, 2 be real numbers such that (1) p,q
Ped=n
08T +cosy +cosz = 1
Waite (1) in the form ¢ + 5 = 1 and use Cauchy-Schwarz inequality to obtain
€03 + cosBy + cos3z = 0.
o
Prove that cos 2z cos 2y cos 2z < 0.
Solution. We shall use the formula cos 3z = 4 cos —3cosz. Summing up
MUPoa wte
2 a
Na
the given equalities yields
that is .
cos®x
+ cos” y + cos® iz
»
From the algebraic decomposition formula The equation of the line PPy is & + £ = 1. The inequality from above shows that
@® 4 b° 46—
Babe = (a+b+c)(4B
a? 4 o — ab— be— ca),
th point @ (4, %) does not lie in the half plane which contains the triangle AP, P.
we deduce cos cosy cos z . Assume cosz = 0. Then —cosy = cosz and 2. Let A,V be 2 x 2 real matrices such that AN = NA and N™ = 0 for
cos 2z = cos2y. Since cos2 ~1,itfollows
that cos 2z cos 2y cos 2z = — cos®
27 < ome positive integer m. Show that
0, and we are done. a) det(4 + V) = det4;
b) If det A 0, then A+ N is invertible and (4 -+ N)~' = (4 = N)4~2,
Solution. Tt is easy to show that if N™ = 0 for some mi > 1, then N? = 0. As
11" GRADE A commutes with I, we have also (NA™)? = 0. The following easy consequence
of the characteristic equation will be used.
1. We are given & rectangle ABCD and let P be an arbitrary point on the Remark. 1f B is 2.2 x 2 real matrix such that B? = 0 then tr(B+1) = 2 and
diagonal BD, P # B,P # D, and Q be an asbitrary point inside the triangle det(B+1) =1
ABD. The perpendicular projections of P on the sides AB, AD are Py, P; respec- Proof. The fact that tr (B 4+ I) = 2 is a consequence of the formula of trace.
tively, a.\al the perpendicular projections of @ on the sides AB, AD are Q1,Qs, The charac equation for B +1 gives
respectively.
Show that if AQ, = } AB and AQa = LAD, then tho point Q doco not lie (I +B)? = 2T + B) +det(I + B) = 0,
inside the triangle AP, Py
from which the conclusion of the remark follows.
Solution. We will prove the statement by using coordinates. Take a sys- In the case the given matrix A is invertible, use the preceding remark for
tem of coordinates such that A(0,0), B(a,0), D(0,b). Consider P of coordinates B=NA"Y. Thus
P(p.q). One has Q (g, ;) det(A+ N) =det (A + NA™!)) = det Adet(I + NA™") = det A.
102 Sowurions N5 - REGIONAL CONTESTS 103
Tn the case when A is not invertible, consider the polynomials f(z) = det(z+ -
4) and g(x) = det(z]+A+N). They are monic and quadratic and since f # 0 and one obtains by subtraction nay 1 — (1= 126 = 20, OF tns = tn "okt >
F(x) = glz) for infinitely many z, it follows that f = 9. In particular £(0) = g(0), 1
This proves that (a,)y is monotonically increasing. Since a1 = (\ ¥ F)Mm
which gives det(A) = det(A + N,
It'is a routine to verify we get
(A=N)A2(A+N)= (A2 - N)A2 = 42472 =1,
which proves (4 — N)A=2 = (A4 N)~1 thus it is bounded and convergent.
3. a) Prove that for all positive integers n, the following inequality holds 4. Let a € (0,1) be a real number and f : R — R be a function which
satisfies the conditions
(o) (1+8) (12 Dot
) <t @) Jim f(z)=0;
0 s,
piCs 2
b) Show that the sequence of real numbers (a),>1 defined by ay = 1 and Show that lim ) — o
2 Solution. From condition (ii) we have: for any € > 0, there exists 6 > 0
Gnsr= =5 Y kay, forall n>1, such that |£(2) — f(az)| < la], for all z € (~5,5). It follows that for all positive
=1 integers 7 and all z € (~,6), one has
is monotonic increasing. Find with proof f it is a convergent sequence. 1f(@) = f(@"2)| < |f(2) ~ f(az)| +|f(az) - f(a2) +e+f(@ ) - f(a)]
Solution. ) We prove the statement by induction on n. For n = 1 we have <elal(l +ata? 4o ot
2 <e, obviously true.
‘Assume that the statement is true for n. To prove it for n+ 1 it suffices to Since & > 0 was arbitrary and £ has limit 0 at zo = 0, it follows that

o) <o
prove that,
17
The inequality is equivalent to 5. Let ABOD be a parallelogram of unequal sides. The point E is the foot
of the perpendicular from B to AC. The line through E which s perpendicular
to BD intersects BC in F and AD in G.
Show that EF = EG if and only if ABCD is a rectangle.
Solution. First, assume that ABCD is a rectangle. Let H be the intersec-
tion point of FG and BD. In the right triangles ABC and FBG, the segments BE
[FREEER L PN° and BH ave altitudes, respectively. Then /ABE = LACB and .BGF = /HBC.
or 1 nin+1)

() Since (HBC = LACB, it follows that /GBE = (BGF and BE = GE. This
proves that GE = EF.
The last is an easy consequence of n(n +1) < (n+1)? and (1 + fi) <e, for B r c B F cL
any positive m.
b) From the equalities 0 Q]
nlangn =Y kax E
A D A 1l D
G
104 Sorumons SoLuTions Recional Contests 105
For the second implication we shall present two solutions. 6. Let A,B be 2-by-2 matrices with integer entries, such that AB = BA
A geometric solution. Let O be the center of the parallelograin. The line. and det B =1
FO intersects AD in I and GI intersects BD in K and BC extended in L. Thus Prove that if det(A® + B%) = 1 then A = I.
EO | GI. Therefore, the quadrilateral GECL is a trapezoid. It is known thiat in a
trapezoid the midpoints of parallel sides and the intersection point of nonparallel Solution. As det B = 1, B is invertible and B! has integer ontrics. From
sides are three collinear points. In our case, it follows that € is the wmidpoint A* 4+ B = ((AB~')+1) BY, it follows that det (AB=1)% +I) = 1. We will
of GL. show that (AB~1)? = 0. Tt will be, thus, sufficient to treat the case B = I.
Consider now the triangle FGL. Since E,K are midpoints of the sides From the decomposition A% + I = (A+ I)(A + e)(A + ¢2I), whore ¢ is
GF,GL respectively, it follows that EK | FL. the complex cubic root of unity, it follows that P(~1)P(~¢)P(%) = 1, where
Finally, we look at the triangle GBK in which GH and BE are altitudes, It P(X) = X? — Tt X + det(4) = X* ~mX +n is the characteristic polynomisl
follows that £ is the orthocenter and EX is perpendicular on GB. Since EX | of A Since P(=£%) = P(~2), it follows that P(—e)P(~£?) € N. Therefore
BC, the conclusion follows. P(-1) = P(—£)P(-£%) =
A solution using coordinates. We formulate the problem as: give o triaugle We get 1+ m + 1 and (€2 + me + m)(e + m: n) = 1, which give
ABC, let BE be the altitude from B and O the midpoint of the side 5C. The =n=0
perpendicular from E on BO is such that the line segments GE and EF are equal.
Then, the angle B is a right angle.
Take the origin of the orthogonal axis to be E and the line EB to be on the
Oy axis. Assume that coordinates of points are A(~a,0), B(0,b),C(e,0) where 12 GRADE
a,b,¢> 0. We have to prove that b? = ac.
1. Let G be a group such that every clement z, & # 1, has order p.
) Show that p is a prime number.
b) Show that if any p? — 1 element subset of G contains p elements which
commute one to another, then G is an Abelian group.
Solution. a) Assume by contradiction that p has the decomposition p = ab,
@>1,b>0. Then, the order of 2 is b, a contradiction.
b) We start with the following
A-TE o c Lemma. Let 2,y € G such that z'y/ = ya’ for all 2 < i,j < p — 1. Then
Go-~ 2y =y
By standard caleulations, we get the following equations and coordinates: Proof. Since (j,p) = 1, there exists a, 1 < a < p 1, such that aj = 1
~line GF: y = <5l (mod p). Then 2'y™) = yz'. Since aj = 1 + mp, y* 412 — y. Therefore
iy =y
~line BC: % In the same way, since (i,p) = 1, one can find b, 1 < b < p 1, such that
=1 (mod p). Since o'y = ya', we deduce 2y = yr* and since bi — 1 = np,
- point F: g Therefore zy = yz, that proves the lemma.
~line AB: ~Z 44 =1 Now, we prove the second statement. Assume by way of contradiction, that
up there exist 2,y € G such that zy # y. Consider the set A C G, A = {a'y? |0 <
- point G: 26 = —g——s,
=52 | ac ey yo = 207+ ac— 4,7 < p— 1)\ {1}. Formally, 4 has p? — 1 elements. By the lemma and the
The condition EG = EF is equivalent to a5v — g, that is assumption it has indeed.
Let B C A be a prelement subso, such that any two clements of B commute.
201 If there exists i, i > 1, such that a* € B, B contains at, least one clement of the
W —actal form 23", 0 < a < p—1,0 < b < p— 1. From 25(a®y?) = (2%}, we get
which easily gives b = ac. a'y® = y*2' and by the lemma, 2y = yz. The same argument shows that if some
element 3 s in B, then zy = yz.
106 SoLuTions
By the previous considerations, we are led to consider that B contains p
elements from the array bellow:

E@ A P ay!

iy el e
By the pigeon-hole principle, there are two clements in B which lie on the same
line. Let they be z°y" a nd 29y, It follows that (z°y*)(z*y") 4*). By
assuming that b > ¢, one has y*~°z® = z%y*~¢. The lemma implies zy = y, a
contradiction.
‘The ROMANIAN MATHEMATICAL SOCIETY (Societatea de Stiinte Matematicé din
Romania) is a centennial professional organization of Romanian mathematician,
1t is the founder, more than hundred years ago of the Romanian Mathematical
Olympiad. The monthly journal *Gazeta Matematics™ is worldwide known as well
as other mathematical texts published by the Society duting yéa
Bttp://wuu. ssuro. con

The THETA FOUNDATION is & non-for-profit organization promoting mathematical


knowledge.
The Theta Foundation is the editor of the “Journal of Operator Theory”. a world-
‘wide known mathematical periodical, and of some important texts in mathematical
research. A series of translations into Romanian of some fundamental monographs
in mathematics is also one of the Theta achievements.
http://uww. theta.to

SOFTWIN is'a noteworthy example of a Romanian software and services


company with extensive experience and great expertise in the education and train-
ing field, particularly in developing intelligent content solutions - such as the In-
tuitext(tm) interactive lessons - meant to improve quality and innovation of the
teaching/learning process in the long run. SOFTWIN's enduring sponsorship of
Romanian students participating in the International Mathematics Olympiads is
another way of supporting and encouraging excellence in the educational environ-
‘ment in Romania.

MEDICOVER
Your personal physician
'SPONSORS OF THE ROMANIAN TEAM FOR THE IMO 2004

ISBN 973-0-03572-5
The ROMANIAN MATHEMATICAL SOCIETY (Societatea de Stiinte Matematice din
Roménia) is a centennial professional organization of Romanian mathematicians.
It is the founder, more than hundred years ago of the Romanian Mathematical
Olympiad. The monthly journal “Gazeta Matematica™ is worldwide known as well
as other mathematical texts published by the Society during vears.
http://www.ssmro.com

The THETA FOUNDATION is a non-for-profit organization promoting mathematical


knowledge.
The Theta Foundation is the editor of the “Journal of Operator Theory™. a world-.
wide known mathematical periodical, and of some important. texts in mathematical
research. A series of translations into Romanian of some fundamental monographs
in mathematics is also one of the Theta achievements.
http://www.theta.ro

SOFTWIN is a noteworthy example of a Romanian software and services


company with extensive experience and great, expertise in the education and train-
ing field, particularly in developing intelligent content solutions - such as the In-
tuitext(tm) interactive lessons - meant to improve quality and innovation of the
teaching/learning process in the long run. SOFTWIN's enduring sponsorship of
Romanian students participating in the International Mathematics Olympiads is
another way of supporting and encouraging excellence in the educational environ-
ment in Romania.

SOFITWIN intuitext enjoy learning !

AN
Y

MEDICOVER
Your personal physician

SPONSORS OF THE ROMANIAN TEAM FOR THE IMO 2004

ISBN 973-0-03572-5

You might also like